Pre-Final

Pataasin ang iyong marka sa homework at exams ngayon gamit ang Quizwiz!

"The nurse is caring for a patient following an appendectomy. The patient takes a deep breath, coughs, and then winces in pain. Which of the following statements, if made by the nurse to the patient, is BEST? "A.) "Take three deep breaths, hold your incision, and then cough." B.) "That was good. Do that again and soon it won't hurt as much." C.) "It won't hurt as much if you hold your incision when you cough." D.) "Take another deep breath, hold it, and then cough deeply."

"(1) correct-most effective way of deep breathing and coughing, dilates airway and expands lung surface area (2) should splint incision before coughing to reduce discomfort and increase efficiency (3) partial answer, should take three deep breaths before coughing (4) implies coughing routine is adequate, incision needs to be splinted"

"The client is diagnosed with Crohn's disease, also known as regional enteritis. Whichstatement by the client would support this diagnosis? (1)."My pain goes away when I have a bowel movement."2."I have bright red blood in my stool all the time."3."I have episodes of diarrhea and constipation."4."My abdomen is hard and rigid and I have a fever."

"(1).The terminal ileum is the most common site for regional enteritis and causes right lower quadrant pain that is relieved by defecation. 2.Stools are liquid or semi-formed and usually do not contain blood.3.Episodes of diarrhea and constipation may be asign/symptom of colon cancer, not Crohn'sdisease.4.A fever and hard rigid abdomen are signs/symptoms of peritonitis, a complication of Crohn's disease."

The nurse is assigning clients for the evening shift. Which of the following clients are appropriate for the nurse to assign to a licensed practical nurse to provide client care? Select all that apply. "1. A client with Crohn's disease who is receiving total parenteral nutrition (TPN). 2. A client who underwent inguinal hernia repair surgery 3 hours ago. 3. A client with an intestinal obstruction who needs a Cantor tube inserted. 4. A client with diverticulitis who needs teaching about his take-home medications. 5. A client who is experiencing an exacerbation of his ulcerative colitis."

"2,5 The nurse should consider client needs and scope of practice when assigning staff to provide care. The client who is recovering from inguinal hernia repair surgery and the client who is experiencing an exacerbation of his ulcerative colitis are appropriate clients to assign to a licensed practical nurse as the care they require fall within the scope of practice for a licensed practical nurse. It is not within the scope of practice for the licensed practical nurse to administer TPN, insert nasoenteric tubes, or provide client teaching related to medications."

"Which of the following diets would be most appropriate for the client with ulcerative colitis? 1. High-calorie, low-protein. 2. High-protein, low-residue. 3. Low-fat, high-fiber. 4. Low-sodium, high-carbohydrate."

"2. High protein, low residue Clients with ulcerative colitis should follow a well-balanced high-protein, high-calorie, low-residue diet, avoiding such high-residue foods as whole-wheat grains, nuts, and raw fruits and vegetables. Clients with ulcerative colitis need more protein for tissue healing and should avoid excess roughage. There is no need for clients with ulcerative colitis to follow low-sodium diets."

"The nurse is teaching the client about gastritis. Which of the following statements by the nurse would be more accurate in describing gastritis? 1. Erosion of the gastric mucosa 2. Inflammation of a diverticulum 3. Inflammation of the gastric mucosa 4. Reflux of stomach acid into the esophagus"

"3. Gastritis is an inflammation of the gastric mucosa that may be accute (often resulting from exposure to local irritants) or chronic (associated w/ autoimmune infections or atrophic disorders of the somach). Erosion of the mucosa results in ulceration. Inflammation of the diverticulum is called diverticulitis; reflux of stomach acid is known as gastroesophageal reflux disease."

"The nurse explains to the patient with gastoesophageal reflux disease that this disorder "a. result in acid erosion and ulceration of the esophagus caused by frequent vomiting. b. will require surgical wrapping or repair of the pyloric sphincter to control the symptoms. c. is the protrusion of a portion of the stomach into the esophagus through an opening in the diaphragm. D. often involves relaxation of the lower esophageal sphincter, allowing stomach contents to back up into the esophagus

"4. Correct answer: d Rationale: Gastroesophageal reflux disease (GERD) results when the defenses of the esophagus are overwhelmed by the reflux of acidic gastric contents into the lower esophagus. An incompetent lower esophageal sphincter (LES) is a common cause of gastric reflux."

"Treatment of Crohn's disease? "1. Diet 2. Vitamins 3. Medications 4. Surgery 5. All of thee above

"A low residue diet is recommend low fiber diet vitamins and iron suplements are recommended Surgery is an option and helps to relieve symptoms (lecture)"

"The results of a patient's recent endoscopy indicate the presence of peptic ulcer disease (PUD). Which of the following teaching points should the nurse provide to the patient in light of his new diagnosis? "A. "You'll need to drink at least two to three glasses of milk daily." B. "It would likely be beneficial for you to eliminate drinking alcohol." C. "Many people find that a minced or pureed diet eases their symptoms of PUD." D. "Your medications should allow you to maintain your present diet while minimizing symptoms.""

"Although there is no specific recommended dietary modification for PUD, most patients find it necessary to make some sort of dietary modifications to minimize symptoms. Milk may exacerbate PUD and alcohol is best avoided because it can delay healing."

A client is admitted with right lower quadrant pain, anorexia, nausea, low-grade fever, and elevated white blood cell count. Which complication is most likely the cause? 1. A. fecalith 2. Bowel Kinking 3. Internal blowel occlusion 4. Abdominal wall swelling

"Answer 1 Rational: The client is experiencing appendicitis. A. fecalith is a fecal calculus, or stone, that occludes the lumen of the appendix and is the most common cause of appendicitis. Bowel wall swelling, kinking of the appendix, and external occlusion not internal occlusion, of the bowel by adhesions can also be cause of appendicitis."

"Which goal for the client's care should take priority during the first days of hospitalization for an exacerbation of ulcerative colitis? source: "A:promoting self care and independence B:managing diarrhea C:maintaining adequate nutrition D:promoting rest and comfort"

"B. managing diarrhea Diarrhea is the primary symptom in an exacerbation of ulcerative colitis, and decreasing the frequency of stools is the first goal of treatment. The other goals are ongoing and will be best achieved by haulting the exacerbation. The client may recieve antidiarrheal agents, antispasmodic agents, bulk hydrophilic agents, or anti-inflammatory drugs."

A client newly diagnosed with Crohn disease asks the nurse whether surgery is necessary to treat the condition. How should the nurse​ respond? ​"Surgery can prevent the​ long-term spread or worsening of Crohn​ disease." ​"Surgery is almost never necessary and cannot treat Crohn disease or its​ complications." ​"Surgery is most often performed when there are complications such as bowel​ obstruction." ​"Surgery is always performed at some point to treat Crohn​ disease."

"Surgery is most often performed when there are complications such as bowel​ obstruction." Bowel obstruction is the leading indication for surgery in Crohn​ disease, which is only performed when necessary to treat or prevent​ complications, or because of the failure of conservative treatments. Surgery is not always​ performed, but it is most often used to treat the complications of the disease. The disease process tends to recur in other areas following removal of affected bowel segments. The risk of fistula formation increases following surgery.

When planning care for a client with ulcerative colitis who is experiencing an exacerbation of symptoms, which client care activities can the nurse appropriately delegate to an unlicensed assistant? Select all that apply. / 1. Assessing the client's bowel sounds. 2. Providing skin care following bowel movements.3. Evaluating the client's response to antidiarrheal medications. 4. Maintaining intake and output records. 5. Obtaining the client's weight.

(2,4, & 5 are CORRECT)The nurse can delegate the following basic care activities to the unlicensed assistant: providing skin care following bowel movements, maintaining intake and output records, and obtaining the client's weight. Assessing the client's bowel sounds and evaluating the client's response to medication are registered nurse activities that cannot be delegated.

13. Which sign/symptoms should the nurse expect to find in a client diagnosed with ulcerative colitis? 1. Twenty bloody stools a day 2. Oral temperature of 102 degrees Fahrenheit 3. Hard, rigid abdomen 4. Urinary stress incontinence

1

17. The client diagnosed with IBD is prescribed total parental nutrition (TPN). Which intervention should the nurse implement? 1. Check the client's glucose level 2. Administer an oral hypoglycemic 3. Assess the peripheral intravenous site 4. Monitor the client's oral food intake

1

22. The client diagnosed with Crohn's disease, also known as regional enteritis. Which statement by the client supports thing diagnosis? 1. "My pain goes away when I have a bowel movement." 2. "I have bright red blood in my stool all the time." 3. "I have episodes of diarrhea and constipation." 4. "My abdomen is hard and rigid and I have a fever."

1

24. The client with ulcerative colitis is scheduled for an ileostomy. The nurse is aware the client's stoma will be located in which area of the abdomen? 1. R Iliac region 2. L Iliac region 3. Epigastric region 4. R Hypochondriac region

1

A client with ulcerative colitis is to take sulfasalazine (Azulfidine). Which of the following instructions should the nurse provide for the client about taking this medication at home? Select all that apply. 1. Drink enough fluids to maintain a urine output of at least 1,200- 1,500 mL per day. 2. Discontinue therapy if symptoms of acute intolerance develop and notify the health care provider. 3. Stop taking the medication if the urine turns orange-yellow. 4. Avoid activities that require alertness. 5. If dose is missed, skip and continue with the next dose.

1, 2, 4. Sulfasalazine may cause dizziness and the nurse should caution the client to avoid driving or other activities that require alertness until response to medication is known. If symptoms of acute intolerance (cramping, acute abdominal pain, bloody diarrhea, fever, headache, rash) occur, the client should discontinue therapy and notify the health care provider immediately. Fluid intake should be sufficient to maintain a urine output of at least 1,200- 1,500 mL daily to prevent crystalluria and stone formation. The nurse can also inform the client that this medication may cause orange-yellow discoloration of urine and skin, which is not significant and does not require the client to stop taking the medication. The nurse should instruct the client to take missed doses as soon as remembered unless it is almost time for the next dose.

The client diagnosed with IBD is prescribed TPN. Which nursing intervention should the nurse implement? (Med-Surg Success, 2nd Edition, Davies Q&A Success Series) 1, Check the patients glucose level 2. Administer and oral hypoglycemic 3. Access the peripheral IV site. 4. Monitor the client's oral food intake

1, Check patient's glucose level

A client with ulcerative colitis expresses serious concerns about her career as an attorney because of the effects of stress on ulcerative colitis. Which of the following nursing interventions will be most helpful to the client? 1. Review her current coping mechanisms and develop alternatives, if needed. 2. Suggest a less stressful career in which she would still use her education and experience. 3. Suggest that she ask her colleagues to help decrease her stress by giving her the easier cases. 4. Prepare family members for the fact that she will have to work part-time.

1. A client with ulcerative colitis need not curtail career goals. Self-care is the cornerstone of long-term management, and learning to cope with and modify stressors will enable the client to live with the disease. Giving up a desired career could discourage and even depress the client. Placing the responsibility for minimizing stressors at work in the hands of others leads to a feeling of loss of control and decreases the sense of responsibility needed for sound self-care. Working part-time rather than full-time is unnecessary.

A client who has had ulcerative colitis for the past 5 years is admitted to the hospital with an exacerbation of the disease. Which of the following factors was most likely of greatest significance in causing an exacerbation of ulcerative colitis? 1. A demanding and stressful job. 2. Changing to a modified vegetarian diet. 3. Beginning a weight-training program. 4. Walking 2 miles every day.

1. Stressful and emotional events have been clearly linked to exacerbations of ulcerative colitis, although their role in the etiology of the disease has been disproved. A modified vegetarian diet or an exercise program is an unlikely cause of the exacerbation.

The client is diagnosed with Crohn's disease, also known as regional enteritis. Which statement by the client supports this diagnosis? 1. "My pain goes away when I have a bowel movement." 2. "I have bright red blood in my stool all the time." 3. "I have episodes of diarrhea and constipation." 4. "My abdomen is hard and rigid and I have a fever."

1. "My pain goes away when I have a bowel movement, 1. (correct) The terminal ileum is the most common site for regional enteritis, which causes right lower quandrant pain that is relieved by defecation. 2. Stools are liquid or semiformed and do not contain blood. 3. Episodes of diarrhea and constipation may be a sign/symptom of colon cancer, not Crohn's disease. 4. A fever and hard rigid abdomen are signs/symptoms of peritonitis, a complication of Crohn's disease.

The client is diagnosed with Crohn's disease, also known as regional enteritis. Which statement by the client would support this diagnosis? 1. "My pain goes away when I have a bowel movement" 2. "I have bright red blood in my stool all the time" 3. "I have episodes of diarrhea and constipation" 4. "My abdomen is hard and rigid and I have a fever".

1. (CORRECT) The terminal ileum is the most common site for regional enteritis and causes right lower quadrant pain that is relieved by defecation 2. Stools are liquid or semi-formed and usually do not contain blood 3. Episodes of diarrhea and constipation may be a sign/symptom of colon cancer, not Crohn'sdisease 4. A fever and hard rigid abdomen are signs/symptoms of peritonitis, a complication of Crohn's disease

The client is diagnosed with an acute exacerbation of ulcerative colitis. Which intervention should the nurse implement? 1. Provide a low-residue diet. 2. Monitor intravenous fluids. 3. Assess vital signs daily. 4. Administer antacids orally.

1. The client's bowel should be placed on rest and no foods or fluids should be introduced into the bowel. 2. (CORRECT) The client requires fluids to help prevent dehydration from diarrheh and to replace fluid lost through normal body functioning. 3. The vital signs must be taken more often than daily in a client who is having an acute exacerbation of ulcerative colitis. 4. The client will receive anti-inflammatory and antidiarrheal medications, not antacids, which are used for gastroenteritis.

What are some common Nursing Interventions to reduce risk/avoid post-op complications?

1. Turn & reposition the pt to promote circulation and reduce the risk of skin breakdown, especially over boney prominences. Initially position pt in a Lateral recumbant position until arousal from anesthesia, then position pt in Semi or Fowler position to reduce breathing effort. 2. Encourage coughing & deep breathing. This helps clear anestetics from the body, lowers risk of pulmonary/fat emboli, and hypostatic pneumonia associated with secretion buildup in the airways. 3. Encourage use of Incentive Spirometer. 4. Monitor In's and Out's. Hydration and protein rich nutrition promotes healing and provides energy to meet the needs of the pts increased metabolism associated with surgery. 5. Promote early ambulation. Early post-op exercise and ambulation significantly reduce the risk of thromboembolism.

15. The client diagnosed with inflammatory bowel disease has a serum potassium level of 3.4 mEq/L. Which action should the nurse implement first? 1. Notify the health-care provider 2. Assess the client fo muscle weakness 3. Request telemetry for the client 4. Prepare to administer potassium IV

2

16. The client is diagnosed with an acute exacerbation of ulcerative colitis. Which intervention should the nurse implement? 1. Provide a low-residue diet 2. Rest the client's bowels 3. Asses vital signs daily 4. Administer antacids orally

2

20. The client diagnosed with ulcerative colitis has an ileostomy. Which statement indicates the client needs more teaching concerning the ileostomy? 1. "My stoma should be pink and moist." 2. "I will irrigate my ileostomy every morning." 3. "If I get a red, bumpy, itchy rash I will call my HCP." 4. "I will change my pouch if it starts leaking."

2

When planning care for a client with ulcerative colitis who is experiencing an exacerbation of symptoms, which client care activities can the nurse appropriately delegate to an unlicensed assistant? Select all that apply. 1. Assessing the client's bowel sounds. 2. Providing skin care following bowel movements. 3. Evaluating the client's response to antidiarrheal medications. 4. Maintaining intake and output records. 5. Obtaining the client's weight.

2, 4, 5. The nurse can delegate the following basic care activities to the unlicensed assistant: providing skin care following bowel movements, maintaining intake and output records, and obtaining the client's weight. Assessing the client's bowel sounds and evaluating the client's response to medication are registered nurse activities that cannot be delegated.

The nurse is developing a plan of care for a client with Crohn's disease who is receiving total parenteral nutrition (TPN). Which of the following interventions should the nurse include? Select all that apply. 1. Monitoring vital signs once a shift. 2. Weighing the client daily. 3. Changing the central venous line dressing daily. 4. Monitoring the I.V. infusion rate hourly. 5. Taping all I.V. tubing connections securely.

2, 4, 5. When caring for a client who is receiving TPN, the nurse should plan to weigh the client daily, monitor the I.V. fluid infusion rate hourly (even when using an I.V. fluid pump), and securely tape all I.V. tubing connections to prevent disconnections. Vital signs should be monitored at least every 4 hours to facilitate early detection of complications. It is recommended that the I.V. dressing be changed once or twice per week or when it becomes soiled, loose, or wet.

The nurse is assigning clients for the evening shift. Which of the following clients are appropriate for the nurse to assign to a licensed practical nurse to provide client care? Select all that apply. 1. A client with Crohn's disease who is receiving total parenteral nutrition (TPN). 2. A client who underwent inguinal hernia repair surgery 3 hours ago. 3. A client with an intestinal obstruction who needs a Cantor tube inserted. 4. A client with diverticulitis who needs teaching about his take-home medications. 5. A client who is experiencing an exacerbation of his ulcerative colitis.

2, 5. The nurse should consider client needs and scope of practice when assigning staff to provide care. The client who is recovering from inguinal hernia repair surgery and the client who is experiencing an exacerbation of his ulcerative colitis are appropriate clients to assign to a licensed practical nurse as the care they require fall within the scope of practice for a licensed practical nurse. It is not within the scope of practice for the licensed practical nurse to administer TPN, insert nasoenteric tubes, or provide client teaching related to medications.

The nurse prepares for the admission of a client with a perforated duodenal ulcer. Which of the following should the nurse expect to observe as the primary initial symptom? 1. Fever 2. Pain 3. Dizziness 4. Vomiting

2, Pain: 1. Fever - later with peritonitis (S/S: pain, nausea, vomiting, rigid abdomen, low-grade fever, absent bowel sounds, shallow respirations). 2. Pain - CORRECT: sudden, sharp, begins mid-epigastric; boardlike abdomen. 3. Dizziness - later with shock (S/S: hypotension, tachycardia, tachypnea, decreases urinary output, decrased LOC). 4. Vomiting - seen with peritonitis

The nurse cares for a client receiving a balanced completed food by tube. The nurse knows the MOST common complication of a tube feeding is which of the following? "1: Edema 2: Diarrhea 3: Hypokalemia 4: Vomiting"

2, diarrhea

Which of the following should be a priority focus of care for a client experiencing an exacerbation of Crohn's disease? 1. Encouraging regular ambulation. 2. Promoting bowel rest. 3. Maintaining current weight. 4. Decreasing episodes of rectal bleeding.

2. A priority goal of care during an acute exacerbation of Crohn's disease is to promote bowel rest. This is accomplished through decreasing activity, encouraging rest, and initially placing client on nothing-by-mouth status while maintaining nutritional needs parenterally. Regular ambulation is important, but the priority is bowel rest. The client will probably lose some weight during the acute phase of the illness. Diarrhea is nonbloody in Crohn's disease, and episodes of rectal bleeding are not expected.

The client with ulcerative colitis is following orders for bed rest with bathroom privileges. When evaluating the effectiveness of this level of activity, the nurse should determine if the client has: 1. Conserved energy. 2. Reduced intestinal peristalsis. 3. Obtained needed rest. 4. Minimized stress.

2. Although modified bed rest does help conserve energy and promotes comfort, its primary purpose in this case is to help reduce the hypermotility of the colon. Remaining on bed rest does not by itself reduce stress, and if the client is having stress, the nurse can plan with the client to use strategies that will help the client manage the stress.

Which of the following diets would be most appropriate for the client with ulcerative colitis? 1. High-calorie, low-protein. 2. High-protein, low-residue. 3. Low-fat, high-fiber. 4. Low-sodium, high-carbohydrate.

2. Clients with ulcerative colitis should follow a well-balanced high-protein, high-calorie, low-residue diet, avoiding such high-residue foods as whole-wheat grains, nuts, and raw fruits and vegetables. Clients with ulcerative colitis need more protein for tissue healing and should avoid excess roughage. There is no need for clients with ulcerative colitis to follow low-sodium diets.

Which goal for the client's care should take priority during the first days of hospitalization for an exacerbation of ulcerative colitis? 1. Promoting self-care and independence. 2. Managing diarrhea. 3. Maintaining adequate nutrition. 4. Promoting rest and comfort.

2. Diarrhea is the primary symptom in an exacerbation of ulcerative colitis, and decreasing the frequency of stools is the first goal of treatment. The other goals are ongoing and will be best achieved by halting the exacerbation. The client may receive antidiarrheal agents, antispasmodic agents, bulk hydrophilic agents, or anti-inflammatory drugs.

A client has been placed on long-term sulfasalazine (Azulfidine) therapy for treatment of his ulcerative colitis. The nurse should encourage the client to eat which of the following foods to help avoid the nutrient deficiencies that may develop as a result of this medication? 1. Citrus fruits. 2. Green, leafy vegetables. 3. Eggs. 4. Milk products.

2. In long-term sulfasalazine therapy, the client may develop folic acid deficiency. The client can take folic acid supplements, but the nurse should also encourage the client to increase the intake of folic acid in his diet. Green, leafy vegetables are a good source of folic acid. Citrus fruits, eggs, and milk products are not good sources of folic acid.

A client with Crohn's disease has concentrated urine, decreased urinary output, dry skin with decreased turgor, hypotension, and weak, thready pulses. The nurse should do which of the following first? 1. Encourage the client to drink at least 1,000 mL per day. 2. Provide parenteral rehydration therapy ordered by the physician. 3. Turn and reposition every 2 hours. 4. Monitor vital signs every shift.

2. Initially, the extracellular fluid (ECF) volume with isotonic I.V. fluids until adequate circulating blood volume and renal perfusion are achieved. Vital signs should be monitored as parenteral and oral rehydration are achieved. Oral fluid intake should be greater than 1,000 mL/ day. Turning and repositioning the client at regular intervals aids in the prevention of skin breakdown, but it is first necessary to rehydrate this client.

A client newly diagnosed with ulcerative colitis who has been placed on steroids asks the nurse why steroids are prescribed. The nurse shuld tell the client? 1. "Ulcerative colitis can be cured by the use of steroids." 2. "Steroids are used in severe flare-ups because they can decrease the incidence of bleeding." 3. "Long-term use of steroids will prolong periods of remission." 4.. "The side effects of steroids outweigh their benefits to clients with ulcerative colitis."

2. Steroids are effective in management of the acute symptoms of ulcerative colitis. Steroids do not cure ulcerative colitis, which is a chronic disease. Long-term use is not effective in prolonging the remission and is not advocated. Clients should be assessed carefully for side effects related to steroid therapy, but the benefits of short-term steroid therapy usually outweigh the potential adverse effects.

The client is diagnosed with an acute exacerbation of ulcerative colitis. Which intervention should the nurse implement? 1.Provide a low-residue diet.2.Monitor intravenous fluids.3.Assess vital signs daily.4.Administer antacids orally.

2. The client requires fluid to prevent dehydration from diarrhea and to replace fluid lost through normal body functioning.

14. The client with type 2 diabetes is prescribed prednisone, a steroid, for an acute exacerbation of inflammatory bowel disease. Which intervention should the nurse discuss with the client? 1. Take this medication on an empty stomach 2. Notify the HCP if experiencing a moon face 3. Take the steroid medication as prescribed 4. Notify the HCP if the blood glucose is over 160

3

18. The client is diagnosed with an acute exacerbation of IBD. Which priority intervention should the nurse implement first? 1. Weigh the client daily and document in the client's chart 2. Teach coping strategies such as dietary modifications 3. Record the frequency, amount, and color of stools 4. Monitor the client's oral fluid intake every shift

3

19. The client diagnosed with Crohn's disease is crying and tells the nurse "I can't take it anymore. I never know when I will get sick and end up here in the hospital." Which statement is the nurse's best response? 1. "I understand how frustrating this must be for you." 2. "You must keep thinking about the good things in your life." 3. "I can see you are very upset. I'll sit down and we can talk." 4. "Are you thinking about doing anything like committing suicide?"

3

23. The client diagnosed with ulcerative colitis is prescribed a low-residue diet. Which meal selection indicates the client understands the diet teaching? 1. Grilled hamburger on a wheat bun and fried potatoes. 2. A chicken salad sandwich and lettuce and tomato salad. 3. Roast pork, white rice, and plain custard. 4. Fried fish, whole grain pasta, and fruit salad.

3

The physician prescribes sulfasalazine (Azulfidine) for the client with ulcerative colitis to continue taking at home. Which instruction should the nurse give the client about taking this medication? 1. Avoid taking it with food. 2. Take the total dose at bedtime. 3. Take it with a full glass (240 mL) of water. 4. Stop taking it if urine turns orange-yellow.

3. Adequate fluid intake of at least 8 glasses a day prevents crystalluria and stone formation during sulfasalazine therapy. Sulfasalazine can cause gastrointestinal distress and is best taken after meals and in equally divided doses. Sulfasalazine gives alkaline urine an orange-yellow color, but it is not necessary to stop the drug when this occurs.

A client's ulcerative colitis signs and symptoms have been present for longer than 1 week. The nurse should assess the client for signs and symptoms of which of the following complications? 1. Heart failure. 2. Deep vein thrombosis. 3. Hypokalemia. 4. Hypocalcemia.

3. Excessive diarrhea causes significant depletion of the body's stores of sodium and potassium as well as fluid. The client should be closely monitored for hypokalemia and hyponatremia. Ulcerative colitis does not place the client at risk for heart failure, deep vein thrombosis, or hypocalcemia.

A client who has ulcerative colitis has persistent diarrhea. He is thin and has lost 12 lb since the exacerbation of his ulcerative colitis. Which of the following will be most effective in helping the client meet his nutritional needs? 1. Continuous enteral feedings. 2. Following a high-calorie, high-protein diet. 3. Total parenteral nutrition (TPN). 4. Eating six small meals a day.

3. Food will be withheld from the client with severe symptoms of ulcerative colitis to rest the bowel. To maintain the client's nutritional status, the client will be started on TPN. Enteral feedings or dividing the diet into six small meals does not allow the bowel to rest. A high-calorie, high-protein diet will worsen the client's symptoms.

A client who has a history of Crohn's disease is admitted to the hospital with fever, diarrhea, cramping, abdominal pain, and weight loss. The nurse should monitor the client for: 1. Hyperalbuminemia. 2. Thrombocytopenia. 3. Hypokalemia. 4. Hypercalcemia.

3. Hypokalemia is the most expected laboratory finding owing to the diarrhea. Hypoalbuminemia can also occur in Crohn's disease; however, the client's potassium level is of greater importance at this time because a low potassium level can cause cardiac arrest. Anemia is an expected development, but thrombocytopenia is not. Calcium levels are not affected.

A client who is experiencing an exacerbation of ulcerative colitis is receiving I.V. fluids that are to be infused at 125 mL/ hour. The I.V. tubing delivers 15 gtt/ mL. How quickly should the nurse infuse the fluids in drops per minute to infuse the fluids at the prescribed rate? ________________________ gtt/ minute.

31 gtt/ minute To administer I.V. fluids at 125 mL/ hour using tubing that has a drip factor of 15 gtt/ mL, the nurse should use the following formula: 125 mL/ 60 minutes × 15 gtt/ 1 mL = 31 gtt/ minute.

21 . The client diagnosed with IBD is prescribed sulfasalazine (Asulfidine), a sulfonamide antibiotic. Which statement best describes the rationale for administering this medication? 1. It is administered rectally to help decrease colon inflammation. 2. This medication slows gastrointestinal motility and reduces diarrhea. 3. This medication kills the bacteria causing the exacerbation. 4. It acts topically on the colon mucosa to decrease inflammation.

4

The nurse has an order to administer sulfasalazine (Azulfidine) 2 g. The medication is available in 500-mg tablets. How many tablets should the nurse administer? ________________________ tablets.

4 tablets To administer 2 g sulfasalazine (Azulfidine), the nurse will need to administer 4 tablets.

"A client is admitted with inflammatory bowel syndrome (Crohn's disease). Which treatment measures should the nurse expect to be part of the care plan? SELECT ALL THAT APPLY! "1) Laculose therapy 2) High fiber diet 3) High protein milkshakes 4) Corticosteroid therapy 5) Antidiarrheal medications

4) Corticosteroid therapy 5) Antidiarrheal medications

A client who has ulcerative colitis says to the nurse, "I can't take this anymore! I'm constantly in pain, and I can't leave my room because I need to stay by the toilet. I don't know how to deal with this." Based on these comments, an appropriate nursing diagnosis for this client would be: 1. Impaired physical mobility related to fatigue. 2. Disturbed thought processes related to pain. 3. Social isolation related to chronic fatigue. 4. Ineffective coping related to chronic abdominal pain.

4. It is not uncommon for clients with ulcerative colitis to become apprehensive and upset about the frequency of stools and the presence of abdominal cramping. During these acute exacerbations, clients need emotional support and encouragement to verbalize their feelings about their chronic health concerns and assistance in developing effective coping methods. The client has not expressed feelings of fatigue or isolation or demonstrated disturbed thought processes.

A client with inflammatory bowel disease (IBD) requires an ileostomy. The nurse would instruct the client to do which of the following measures as an essential part of caring for the stoma? 1.Perform massage of the stoma three times a day. 2.Include high-fiber foods in the diet, especially nuts. 3.Limit fluid intake to prevent loose stools. 4.Cleanse the peristomal skin meticulously.

4; cleanse peristomal skin meticulously1.It is not an intervention used for ileostomies. 2.Clients should avoid the high-fiber and gas-producing foods. 3.These clients are not on fluid restriction. 4.Careful cleansing is necessary to prevent skin breakdown and skin irritation.

A patient is newly diagnosed with mild ulcerative colitis. What type of anti-inflammatory medication is typically prescribed as first-line treatment for this condition?* A. 5-Aminosalicylates (Sulfasalazine) B. Immunomodulators (Adalimumab) C. Corticosteroids (Prednisone) D. Immunosupressors (Azathioprine)

A

A patient with Crohn's Disease is taking corticosteroids. The patient is complaining of extreme thirst, polyuria, and blurred vision. What is your next nursing action? A. Check the patient's blood glucose B. Give the patient a food containing sugar (ex: orange juice) C. Administer oxygen via nasal cannula D. Assess bowel sounds

A

A patient with ulcerative colitis is scheduled for ileoanal anastomosis (J-Pouch) surgery. You know that this procedure:* A. Removes the colon and rectum which allows a pouch to be created that will attach to the ileum. This will allow stool to pass from the small intestine to the anus. B. Removes the colon and rectum and creates a permanent ileostomy. C. Removes the colon and creates a temporary colostomy. D. Removes the rectum which allows a pouch to be created from the colon. This will allow stool to pass from the colon to the anus.

A

The nurse performs a detailed assessment of the abdomen of a patient with a possible bowel obstruction, knowing that a manifestion of an obstruction in the large intestine is (select all that apply) a ) a largely distended abdomen b) diarrhea that is loose or liquid c) persistent, colickcy abdominal pain d) profuse vomiting that relieves abdominal pain.

A & C: distended abd + colicky abd painPersistent, colicky abdominal pain is seen with lower intestinal obstruction. Abdominal distention is markedly incerased in lower interestinal obstructions. Onset of a large intestine obstruction is gradual, vomiting is rare, and there is usually absolute constipation.

Endoscopy

A medical procedure that uses an instrument called an endoscope. Chapter 46; Page 1091

Wound Ostomy Continence Nurse (WOCN)

A nurse specially educated to care for ostomy patients; the nurse collaborates with staff nurses to be use that the patient uses the correct pouching system especially when the patient is ill or is experiencing health changes or problems with the ostomy. Chapter 46; Page 1109

Bowel Training

A program helps some patients defecate normally, especially those who still have some neuromuscular control. Chapter 46; Page 1110

The nurse is preparing to send a patient to the operating room for an exploratory laparoscopy. The nurse recognizes that there is no informed consent for the procedure on the patient's chart. The nurse informs the physician who is performing the procedure. The physician asks the nurse to obtain the informed consent signature from the patient. The nurse's best action to the physician's request is to: A. Inform the physician that is his responsibility to obtain the signature. B. Obtain the signature and ask another nurse to co-sign the signature. C. Inform the physician that the nurse manager will need to obtain the signature.

A) The responsibility for securing informed consent from the patient lies with the person who will perform the procedure. The nurse's best action is to inform the physician that it is his responsibility to obtain the signature.

You're providing diet teaching to a patient with ulcerative colitis about what types of foods to avoid during a "flare-up". Which foods below should the patient avoid? SELECT-ALL-THAT-APPLY:* A. Ice cream B. White Rice C. Fresh apples and pears D. Popcorn E. Cooked carrots

A, C, D

A patient with a history of peptic ulcer disease has presented to the emergency department with complaints of severe abdominal pain and a rigid, boardlike abdomen, prompting the health care team to suspect a perforated ulcer. Which of the following actions should the nurse anticipate? a. Providing IV fluids and inserting a nasogastric tube; b. Administering oral bicarbonate and testing the patient's gastric pH level; c. Performing a fecal occult blood test and administering IV calcium gluconate; d. Starting parenteral nutrition and placing the patient in a high-Fowler's position;

A, IV fluids + NG tubeA perforated peptic ulcer requires IV replacement of fluid losses and continued gastric aspiration by NG tube. Nothing is given by mouth and gastric pH testing is not a priority. Calcium gluconate is not a medication directly relevant to the patient's suspected diagnosis and parenteral nutrition is not a priority in the short term

"The nurse is teaching about irritable bowel syndrome. Which of the following would be most important? "A. Reinforcing the need for a balanced diet B. Encouraging the client to drink 16 ounces of fluid with each meal C. Telling the client to eat a diet low in fiber D. Instructing the client to limit his intake of fruits and vegetables"

A, reinforce the need for balanced dietThe nurse should reinforce the need for a diet balanced in all nutrients and fiber. Foods that often cause diarrhea and bloating associated with irritable bowel syndrome include fried foods, caffeinated beverages, alcohol, and spicy foods. Therefore, answers B, C, and D are incorrect.

Select ALL of the following that are complications associated with Crohn's Disease: A. Cobble-stone appearance of GI lining B. Lead-pipe sign C. Toxic megacolon D. Fistula E. Abscess F. Anal Fissure

A,D,E,F

The reason pts are sent to a PACU after surgery is: A. to be monitored while recovering from anesthesia. B. to remain near the surgeon immediately after surgery. C. to allow the medical-surgical unit time to prepare for transfer. D. to provide time for the pt to cope with the effects of surgery.

A. Pts are sent to a PACU to be monitored while they're recovering from anesthesia.

In the operating room, a patient tells a circulating nurse that he is going to have the cataract in his left eye removed. If the nurse notes that the consent form indicates that surgery is to be performed on the right eye, what should be the nurse's first action? A. Ask the patient his name. B. Notify the surgeon and anesthesiologist. C. Check to see whether the patient has received any preoperative medications. D. Assume that the patient is a little confused because he is older and has received midazolam intramuscularly.

A. Ask the patient his name. Ensuring proper identification of a patient is a responsibility of all members of the surgical team. In a specialty surgical setting where many patients undergo the same type of surgery each day, such as cataract removal, it is possible that the patient and the record do not match. Nurses do not assume in the care of their patients. The priority is with the nurse identifying the patient and the patient's consent form before the physicians are notified.

A nurse is caring for a client for whom a tap water enema is prescribed, to be repeated until the return is clear. Which of the following actions should the nurse take? A. Clarify the order with the provider. B. Explain the procedure to the client. C. Ensure that the tap water is not too hot. D. Keep the amount per enema to less than 1,000 mL.

A. Clarify the order with the provider. Tap water is a hypotonic solution that can cause water toxicity. It should not be repeated. The nurse should clarify the order with the provider. Explaining the procedure to the client, ensuring that the tap water is not too hot, and keeping the amount to less than 1,000 mL are not pertinent if the enema should not be repeated. If you got this question wrong, I am judging you...just kidding :)

A patient is suspected to have diverticulosis without symptoms of diverticulitis. What diagnostic test does the nurse anticipate educating the patient about prior to discharge? A. Colonoscopy B. barium enema C. flexible sigmoidoscopy D. CT scan

A. Colonoscopy

A patient is having a diagnostic work up for reports of frequent diarrhea, right lower abdominal pain, and weight loss. The nurse is reviewing the results of the barium study and notes the presence of "string sign." What does the nurse recognize that this is significant of? A. Crohn's disease B. ulcerative colitis C. irritable bowel syndrome D. diverticulitis

A. Crohn's disease

Select all that apply. A nurse is caring for a surgical patient in the preoperative area. The nurse obtains the patient's informed consent for the surgical procedure. Which statements are true regarding informed consent? A. Informed consent must be signed while the patient is free from mind-altering medications. B. Informed consent must be witnessed. C. Informed consent may be withdrawn at any time. D. Informed consent must be signed by patients age 16 and older. E. Informed consent must be obtained by the physician. F. Informed consent must be obtained from the family even in a life-threatening emergency.

A. Informed consent must be signed while the patient is free from mind-altering medications. B. Informed consent must be witnessed. An informed consent must be signed while the patient is free from mind-altering medications and must be witnessed after it has been determined that the patient has received all of the necessary information needed to make an informed decision. An informed consent may be withdrawn at any time before the procedure and must be signed by patients age 18 and older. A parent or guardian's signature is required for minors. The informed consent may be obtained by the physician or the nurse and is not required in the event of a life-threatening emergency.

The nurse is caring for an older adult patient experiencing fecal incontinence. When planning the care of this patient, what should the nurse designate as a priority goal? A. Maintaining skin integrity B. beginning a bowel program to establish continence C. instituting a diet high in fiber and increase fluid intake D. determining the need for surgical intervention to correct the problem

A. Maintaining skin integrity

Select all that apply. Which of the following best describes a consent form? A. May be signed by an emancipated minor. B. Protects the health care facility but not the physician C. Signifies that the patient understands all aspects of the procedure. D. Signifies that the patient and family have been told about the procedure E. Must be signed by the patient or responsible party at the health care facility, and that consent may not be obtained by phone or fax

A. May be signed by an emancipated minor. (&) C. Signifies that the patient understands all aspects of the procedure. A consent form may be signed by an emancipated minor, and consent may be obtained by fax or phone with appropriate witnesses. Only in the cases of underage children or unconscious or mentally incompetent people must a family member be aware of the procedure. The document protects the surgeon and the health care facility in that it indicates that the patient knows and understands all aspects of the procedure.

Which of the following nursing interventions should receive highest priority when a patient is admitted to the postanesthesia care unit? A. Positioning the patient B. Observing the operative site C. Checking the postoperative orders D. Receiving report from operating room personnel.

A. Positioning the patient. A patient is received in the postanesthesia care unit on a bed or stretcher. Proper positioning is necessary to ensure airway patency in a sedated, unconscious, or semiconscious patient. Observation of the operative site, receiving report from operating room personnel, and checking postoperative orders are interventions made after proper positioning of the patient.

Diarrhea

An increase in the number of stools and the passage of liquid, unformed feces. Chapter 46; Page 1092

The nurse is reviewing the record of a female client with Crohn's disease. Which stool characteristics should the nurse expect to note documented in the client's record 1. Diarrhea 2. Chronic constipation 3. Constipation alternating with diarrhea 4. Stools constantly oozing form the rectum

Answer 1: Diarrhea, Crohn's disease is characterized by nonbloody diarrhea and around 4-5 stools per day. Over time, episodes of diarrhea increase in frequency, duration, and severity.

"A nurse is caring for a client admitted to the hospital with a suspected diagnosis of acute appendicitis. Which of the following laboratory results would the nurse expect to note if the client does have appendicitis? 1. Leukopenia with a shift to the right 2. Leukocytosis with a shift to the right 3.Leukocytosis with a shift to the left 4. Leukopenia with a shift to the left"

Answer 2 - no rationale

"In planning care for the patient with crohns disease the nurse recognizes that a major difference between UC and Crohn's disease is that: 1. Frequently results in toxic megacolon 2. causes fewer nutritional deficiencies than does UC 3. Often recurs after surgery whereas UC is curable with a colectomy 4. is manifested by rectal bleeding and anemia more frequently than UC"

Answer 3, "Medication is the primary treatment for Crohn's disease"

The client diagnosed with Crohn's disease is crying and tells the nurse, "I can't take itanymore. I never know when I will get sick and end up here in the hospital." Whichstatement would be the nurse's best response? 1."I understand how frustrating this must be for you."2."You must keep thinking about the good things in your life." 3. "I can see you are very upset. I'll sit down and we can talk."4."Are you thinking about doing anything like committing suicide?"

Answer 3, I can see you are very upset, I'll sit down and we can talk, "1.The nurse should never tell a client that they understand what they are going through.2.This is not addressing the client's feelings. 3. (Correct answer) The client is crying and is expressing feel-ings of powerlessness; therefore the nurseshould allow the client to talk. 4.The client is crying and states "I can't take itanymore," but this is not a suicidal commentor situation."

18. The client is diagnosed with an acute exacerbation of IBD. Which priority intervention should the nurse implement first? "1. Weigh the client daily and document it in the client's chart. 2. Teach coping strategies such as dietary modifications. 3. Record the frequency, amount, and color of stools. 4. Monitor the client's oral fluid intake every shift."

Answer 3, Record the frequency, amount, and color of stools"Rationale by answer option: 1. Weighing the client daily will help identify if the client is experiencing malnutrition, but it is not the priority intervention during an acute exacerbation. 2. Coping strategies help develop healthy ways to deal with this chronic disease that has remissions and exacerbations, but it is not the priority intervention. 3. The severity of the diarrhea helps determine the need for fluid replacement. The liquid stool should be measured as part of the total output. (CORRECT) 4. The client will be NPO when there is an acute exacerbation of IBD to allow the bowel to rest."

"Which associated disorder might a client with ulcerative colitis exhibit "1. Gallstone 2. Hyronephrosis 3.Nephrolithiasis 3. Toxic megacolon

Answer 3, Toxic megacolon is extreme dilation of a segemnt of the diseased colon caused by paralysis of the colon

A patient returns to his room following a lower GI series. When he is assessed by the nurse, he complains of weakness. Which of the following nursing diagnoses should receive priority in planning his care? 1. Alteration in sensation - gustatory 2. Constipation, colonic 3. High risk for fluid volume deficit 4. Nutrition, less than body requirements

Answer 3, high risk for fluid vol. deficitprep for test: low-residue or clear liquid diet 2 days, NPO midnight, enemas, laxatives, post-test: laxatives to remove barium

The nurse is caring for a client diagnosed with ulcerative colitis. Which symptom(s) support this diagnosis? 1. Increased appetite and thirst 2. Elevated hemoglobin 3. Multiple bloody, liquid stools. 4. Exacerbations unrelated to stress

Answer 3: Clients report as many as 10 to 20 liquid bloody stools in a day.

Which associated disorder might a client with Crohn's disease exhibit most often? 1. Ankylosing spondylitis 2. Colon cancer 3. Malabsorption 4. Lactase deficiency

Answer 3; Malabsorption Because of the transmural nature of Crohn's disease lesions, malabsorption may occur with Crohn's disease. Although ankylosing spondylitis and colon cancer are more commonly associated with ulcerative colitis, they may be seen in clients with Crohn's disease, Lactase deficiency is caused by a congenital defect in which an enzyme isn't present.

"Which sign/symptom should the nurse expect to find in a client diagnosed with ulcerative colitis? 1. Twenty bloody stools a day. 2. Oral temperature of 102 ̊F. 3. Hard, rigid abdomen. 4. Urinary stress incontinence."

Answer = 1. The colon is ulcerated and unable to absorb water, resulting in bloody diar- rhea. Ten (10) to 20 bloody diarrhea stools is the most common symptom of ulcerative colitis. 2. Inflammation usually causes an elevated temperature but is not expected in the client with ulcerative colitis. 3.A hard, rigid abdomen indicates peritonitis, which is a complication of ulcerative colitis but not an expected symptom. 4. Stress incontinence is not a symptom of colitis."

"The nurse is caring for a patient in the emergency department with complaints of acute abdominal pain, nausea, and vomiting. When the nurse palpates the patient's left lower abdominal quadrant, the patient complains of pain in the right lower quadrant. The nurse will document this as which of the following diagnostic signs of appendicitis? "a. Rovsing sign b. referred pain c. Chvostek's sign d. rebound tenderness correct answer: A"

Answer A In patients with suspected appendicitis, Rovsing sign may be elicited by palpation of the left lower quadrant, causing pain to be felt in the right lower quadrant.

The nurse is teaching a female client how to perform a colostomy irrigation. To enhance the effectiveness of the irrigation and fecal returns, what measure would the nurse tell the client to do? "a. Increase fluid intake b. Place heat on the abdomen c. Perform the irrigation in the evening d. Reduce the amount of irrigation solution"

Answer A. To enhance effectiveness of the irrigation and fecal returns, the client is instructed to increase fluid intake and to take other measures to prevent constipation. Options B, C and D will not enhance the effectiveness of this procedure.

What is one of the major precipitating factors in the development of irritable bowel syndrome (IBS)? "A. Stress B. Peptic ulcers C. GERD D.Helicobacter pylori"

Answer A: Stress, Rationale: Stress is one of the major factors for developing irritable bowel syndrome (IBS), along with dietary factors.

"When preparing a male client, age 51, for surgery to treat appendicitis, the nurse formulates a nursing diagnosis of Risk for infection related to inflammation, perforation, and surgery. What is the rationale for choosing this nursing diagnosis? "a. Obstruction of the appendix may increase venous drainage and cause the appendix to rupture. b. Obstruction of the appendix reduces arterial flow, leading to ischemia, inflammation, and rupture of the appendix. c. The appendix may develop gangrene and rupture, especially in a middle-aged client. d. Infection of the appendix diminishes necrotic arterial blood flow and increases venous drainage."

Answer B. A client with appendicitis is at risk for infection related to inflammation, perforation, and surgery because obstruction of the appendix causes mucus fluid to build up, increasing pressure in the appendix and compressing venous outflow drainage. The pressure continues to rise with venous obstruction; arterial blood flow then decreases, leading to ischemia from lack of perfusion. Inflammation and bacterial growth follow, and swelling continues to raise pressure within the appendix, resulting in gangrene and rupture. Geriatric, not middle-aged, clients are especially susceptible to appendix rupture.

The nurse is caring for a male client postoperatively following creation of a colostomy. Which nursing diagnosis should the nurse include in the plan of care? -1) a. sexual dysfunction b. body image, disturbed c. fear related to poor prognosis d. Nutrition: more than body requirements, imbalanced

Answer B. Body image, disturbed relates to loss of bowel control, the presence of a stoma, the release of fecal material onto the abdomen, the passage of flatus, odor, and the need for an appliance (external pouch). No data in the question support options A and C. Nutrition: less than body requirements, imbalanced is the more likely nursing diagnosis.

The doctor ordered for a complete blood count. After the test, Nurse Ray received the result from the laboratory. Which laboratory values will confirm the diagnosis of appendicitis? a. RBC 5.5 x 106/mm3 b. Hct 44 % c. WBC 13, 000/mm3 d. Hgb 15 g/dL"

Answer C "Rationale: Increase in WBC counts is suggestive of appendicitis because of bacterial invasion and inflammation. Normal WBC count is 5, 000 - 10, 000/mm3. Other options are normal values."

THe nurse is caring for a client with a diagnosis of Crohn's disease. When evaluating a clients response to healthcare intervention, which expected outcome is the most important for the client: A. does skincare B. takes oral fluids C. gains .5 lb per week D. experiences less abdominal cramping

Answer C = gains .5lbs/wk, weight loss usually is severe with Crohn's disease, therefore, weight gain is a priority. this goal is specific, realistic, measureable and has a timeframe.

In planning care for the patient with Crohn's disease, the nurse recognizes that a major difference between ulcerative colitis and Crohn's disease is that Crohn's disease: (Source: Medical-Surgical Nursing, LHD pg. 1055) A. frequently results in toxic megacolon, B. causes fewer nutritional deficiencies than does ulcerative colitis, C. often recurs after surgery, whereas ulcerative colitis is curable with a colectomy, D. is manifested by rectal bleeding and anemia more frequently than is ulcerative colitis.

Answer C, often recurs after surgery, whereas ulcerative colitis is curable with a colectomyRationale: Because there is a high recurrence rate after surgical treatment of Crohn's disease, medications are the preferred treatment.

The nurse is caring for a hospitalized female client with a diagnosis of ulcerative colitis. Which finding, if noted on assessment of the client, would the nurse report to the physician? "a. Hypotension b. Bloody diarrhea c. Rebound tenderness d. A hemoglobin level of 12 mg/dL"

Answer C. Rebound tenderness may indicate peritonitis. Bloody diarrhea is expected to occur in ulcerative colitis. Because of the blood loss, the client may be hypotensive and the hemoglobin level may be lower than normal. Signs of peritonitis must be reported to the physician.

During the assessment of a patient with acute abdominal pain, the nurse should: a. perform deep palpation before ausculation b. obtain BP and pulse rateto determine hypervolemic changes c. auscultate bowel sounds because hyperactive bowel sounds suggest paralytic ileus d. measure body temperature because and elevated temperature may indicate an inflammatory or infectious process.

Answer D, If the temperature is elevated pain may be due to infection.

A nurse is caring for a child who had a laproscopic appendectomy. What interventions should the nurse document on the child's clinical record? Select all that apply. 1) Intake and Output 2) Measurement of Pain 3) Tolerance to low-residue diet 4) Frequency of dressing changes 5) Auscultation of bowel sounds

Answer: 1, 2, 5 1) Assessment and documentation of fluid balance are critical aspects of all postoperative care. 2) Laparoscopic surgery involves insufflating the abdominal cavity with air, which is painful until it is absorbed. The amount of pain should be measured and documented with either a 1-10 scale or the Wong's FACES for younger children. 3) A special diet is not indicated after this surgery. 4) After a laparoscopic appendectomy there is little drainage and no dressings. 5) Auscultating for bowel sounds and documenting their presennce or absence evaluate the child's adaptation to the intestinal trauma caused by the surgery.

The nurse is assessing an adolescent who is admitted to the hospital with appendicitis. The nurse should report which of the following to the HCP? "1) change in pain rating of 7 to 8 on a 10 point scale. 2) sudden relief of sharp pain, shifting to diffuse pain. 3)shallow breathing with normal vital signs. 4) decrease of pain rating from 8 to 6 when parents visit.

Answer: 2 Rationale: The nurse notifies the HCP if the client has sudden relief of sharp pain and on presence of more diffuse pain. this change in the pain indicates the appendix has ruprured. The diffuse pain is typically accompanied by rigid guarding of the abdomen, progressive abdominal distension, tachycardia, pallor, chills, and irritability. The slight increase pain can be expected; the decrease in pain when parents visit may be attributed to being distracted from the pain. shallow breathing is likely due to the pain and is insignificant when other vital signs are normal

A school-aged child has an emergency appendectomy. The nurse should report which of the following to the HCP if notes in the immediate postoperative period. 1. abdominal pain, 2. tugging at the incision line, 3. thirst, 4 a rigid abdomen

Answer: 4 Rationale: A tense, rigid abdomen is an early symptom of peritonitis. The other findings are expected in the immediate postoperative period.

After admission of the postoperative patient to the clinical unit, which assessment data requires the most immediate attention? A) Oxygen saturation of 85% B) Respiratory rate of 13/minute C) Temperature of 100.4F (38 C) D) Blood pressure of 90/60

Answer: A During the initial assessment, identify signs of inadequate oxygenation and ventilation. Pulse oximetry monitoring is initiated because it provides a noninvasive means of assessing the adequacy of oxygenation. Pulse oximetry may indicate low oxygen saturation (<90% to 92%) with respiratory compromise. This necessitates prompt intervention.

In planning postoperative interventions to promote repositioning, ambulation, coughing, and deep breathing, which action should the nurse recognize will best enable the patient to achieve the desired outcomes? A) Administering adequate analgesics to promote relief or control of pain. B) Asking the patient to demonstrate the postoperative exercises every 1 hour C) Giving the patient positive feedback when the activities are performed correctly D) Warning the patient about possible complications if the activities are not performed

Answer: A Even when a patient understands the importance of postoperative activities and demonstrates them correctly, it is unlikely that the best outcome will occur unless the patient has sufficient pain relief to cooperate with the activities.

The nurse is caring for a 54-year-old unconscious female patient who has just been admitted to the postanesthesia care unit after abdominal hysterectomy. How should the nurse position the patient? A) Left lateral position with head supported on a pillow B) Prone position with a pillow supporting the abdomen C) Supine position with head of bed elevated 30 degrees D) Semi-Fowler's position with the head turned to the right

Answer: A The unconscious patient should be placed in the lateral "recovery" position to keep the airway open and reduce the risk of aspiration. Once conscious, the patient is usually returned to a supine position with the head of the bed elevated to maximize expansion of the thorax by decreasing the pressure of the abdominal contents on the diaphragm.

Which of the following complications is thought to be the most common cause of appendicitis? a. A fecalith b. Internal bowel occlusion c. Bowel kinking d. Abdominal wall swelling"

Answer: A. A fecalith Rationale: A fecalith is a hard piece of stool which is stone like that commonly obstructs the lumen. Due to obstruction, inflammation and bacterial invasion can occur. Tumors or foreign bodies may also cause obstruction."

The nurse is working on a surgical floor and is preparing to receive a postoperative patient from the postanesthesia care unit (PACU). What should the nurse's initial action be upon the patient's arrival? A) Assess the patient's pain. B) Assess the patient's vital signs. C) Check the rate of the IV infusion. D) Check the physician's postoperative orders

Answer: B The highest priority action by the nurse is to assess the physiologic stability of the patient. This is accomplished in part by taking the patient's vital signs. The other actions can then take place in rapid sequence.

The patient had abdominal surgery. The estimated blood loss was 400 mL. The patient received 300 mL of 0.9% saline during surgery. Postoperatively, the patient is hypotensive. What should the nurse anticipate for this patient? A) Blood administration B) Restoring circulating volume C) An ECG to check circulatory status D) Return to surgery to check for internal bleeding

Answer: B The nurse should anticipate restoring circulating volume with IV infusion. Although blood could be used to restore circulating volume, there are no manifestations in this patient indicating a need for blood administration. An ECG may be done if there is no response to the fluid administration, or there is a past history of cardiac disease, or cardiac problems were noted during surgery. Returning to surgery to check for internal bleeding would only be done if patient's level of consciousness changes or the abdomen becomes firm and distended.

Unless contraindicated by the surgical procedure, which position is preferred for the unconscious patient immediately postoperative? A) Supine B) Lateral C) Semi-Fowler's D) High-Fowler's

Answer: B Unless contraindicated by the surgical procedure, the unconscious patient is positioned in a lateral "recovery" position. This recovery position keeps the airway open and reduces the risk of aspiration if the patient vomits. Once conscious, the patient is usually returned to a supine position with the head of the bed elevated.

When a patient is admitted to the PACU, what are the priority interventions the nurse performs? A) Asses the surgical site, noting presence and character of drainage B) Assess the amount of urine output and the presence of bladder distention C) Assess for airway patency and quality of respirations, and obtain vital signs D) Review results of intraoperative laboratory values and medications received.

Answer: C Assessment in the postanesthesia care unit (PACU) begins with evaluation of the airway, breathing, and circulation (ABC) status of the patient. Identification of inadequate oxygenation and ventilation or respiratory compromise necessitates prompt intervention.

A patient is admitted to the PACU after major abdominal surgery. During the initial assessment the patient tells the nurse he thinks he is going to "throw up." A priority nursing intervention would be to: A) increase the rate of the IV fluids. B) obtain vital signs including O2 saturation. C) position patient in lateral recovery position. D) administer antiemetic medication as ordered.

Answer: C If the patient is nauseated and may vomit, place the patient in a lateral recovery position to keep the airway open and reduce the risk of aspiration if vomiting occurs.

In caring for the postoperative patient on the clinical unit after transfer from the PACU, which care can be delegated to the unlicensed assistive personnel (UAP)? A) Monitor the patient's pain. B) Do the admission vital signs. C) Assist the patient to take deep breaths and cough. D) Change the dressing when there is excess drainage.

Answer: C The UAP can encourage and assist the patient to do deep breathing and coughing exercises and report complaints of pain to the nurse caring for the patient. The RN should do the admission vital signs for the patient transferring to the clinical unit from the PACU. The LPN or RN will monitor and treat the patient's pain and change the dressings.

When assessing a patient's surgical dressing on the first postoperative day, the nurse notes new, bright-red drainage about 5 cm in diameter. In response to this finding, what should the nurse do first? A) Recheck in 1 hour for increased drainage. B) Notify the surgeon of a potential hemorrhage. C) Assess the patient's blood pressure and heart rate. D) Remove the dressing and assess the surgical incision.

Answer: C The first action by the nurse is to gather additional assessment data to form a more complete clinical picture. The nurse can then report all of the findings. Continued reassessment will be done. Agency policy determines whether the nurse may change the dressing for the first time or simply reinforce it.

A 67-year-old male patient is admitted to the postanesthesia care unit (PACU) after abdominal surgery. Which assessment, if made by the nurse, is the best indicator of respiratory depression? A) Increased respiratory rate B) Decreased oxygen saturation C) Increased carbon dioxide pressure D) Frequent premature ventricular contractions (PVCs)

Answer: C Transcutaneous carbon dioxide pressure (PtcCO2) monitoring is a sensitive indicator of respiratory depression. Increased CO2 pressures would indicate respiratory depression. Clinical manifestations of inadequate oxygenation include increased respiratory rate, dysrhythmias (e.g., premature ventricular contractions), and decreased oxygen saturation.

A client with complaints of right lower quadrant pain is admitted to the emergency department. Blood specimens are drawn and sent to the laboratory. Which laboratory finding should be reported to the physician immediately? "a) Hematocrit 42% b) Serum potassium 4.2 mEq/L c) Serum sodium 135 mEq/L d) White blood cell (WBC) count 22.8/mm3.

Answer: D "D) White blood cell (WBC) count 22.8/mm3 The nurse should report the elevated WBC count. This finding, which is a sign of infection, indicates that the client's appendix might have ruptured. Hematocrit of 42%, serum potassium of 4.2 mEq/L, and serum sodium of 135 mEq/L are within normal limits. Alterations in these levels don't indicate appendicitis."

A 70kg postoperative patient has an average urine output of 25mL/hr during the first 8 hours. The priority nursing intervention(s) given this assessment would be to: A) perform a straight catheterization to measure the amount of urine in the bladder. B) notify the physician and anticipate obtaining blood work to evaluate renal function. C) continue to monitor the patient because this is a normal finding during this time period. D) evaluate the patient's fluid volume status since surgery and obtain a bladder ultrasound.

Answer: D Because of the possibility of infection associated with catheterization, the nurse should first try to validate that the bladder is full. The nurse should consider fluid intake during and after surgery and should determine bladder fullness by percussion, by palpation, or by a portable bladder ultrasound study to assess the volume of urine in the bladder and avoid unnecessary catheterization.

Bobby, a 13 year old is being seen in the emergency room for possible appendicitis. An important nursing action to perform when preparing Bobby for an appendectomy is to:""a) administer saline enemas to cleanse the bowels b) apply heat to reduce pain c) measure abdominal girth d) continuously monitor pain

Answer: D Rationale: Pain is closely monitored in appendicitis. In most cases, pain medication is not given until prior to surgery or until the diagnosis is confirmed to be able to closely monitor the progression of the disease. A sudden change in the character of pain may indicate rupture or bowel perforation. Administering an enema or applying heat may cause perforation and abdominal girth may not change with appendicitis.

Paralytic ileus

Any surgery that involves direct manipulation of the bowel temporarily stops peristalsis. Chapter 46; Page 1091

The nurse is caring for a client in the early stages of Crohn disease. Which type of lesion should the nurse recall that occurs when at the beginning of this disease​ process? Aphthoid Crypt abscess Canker sore Fistula

Aphthoid Aphthoid lesions are​ small, inflammatory ulcers with a white base and elevated margin. They have a similar appearance to a canker​ sore, but they are not actually cankers. A crypt abscess is found in the beginning stages of ulcerative​ colitis, not Crohn disease. Fistulas appear as Crohn disease​ progresses, not in the early stages.

Polyps

Are abnormal growths of tissue that can be found in any organ that has blood vessels. Chapter 46; Page 1094

Flatulence

As gas accumulates in the lumen of the intestines, the bowel wall stretches and distends. Chapter 46; Page 1092

You're providing education to a patient with severe ulcerative colitis about Adalimumab. Which statement by the patient is CORRECT?* A. "This medication is used as first-line treatment for ulcerative colitis." B. "My physician will order a TB skin test before I start taking this medication." C. "This medication works by increasing the tumor necrosis factor protein which helps decrease inflammation." D. "This medication is a corticosteroid. Therefore, I need to monitor my blood glucose levels regularly."

B

A patient returning to the floor after orthopedic surgery is complaining of nausea. The nurse is aware that an appropriate intervention is to: A. Hold all medications. B. Avoid strong smelling foods. C. Avoid oral hygiene until the nausea subsides. D. Provide clear liquids with a straw.

B)Nursing care for a patient with nausea includes avoiding strong smelling foods. Providing oral hygiene, administering prescribed medications (especially if medications ordered are anti-nausea/antiemetics), and avoid the use of a straw.

You're educating a group of outpatients about signs and symptoms of ulcerative colitis. Which of the following are NOT typical signs and symptoms of ulcerative colitis? SELECT-ALL-THAT-APPLY:* A. Rectal Bleeding B. Abdominal mass C. Bloody diarrhea D. Fistulae E. Extreme Hungry F. Anemia

B, D, E

Older patients with longstanding or severe Crohn's disease can exhibit which of the following? A. Hyperalbumineria B. Hypoalbumenria C. Decreased Sedimentation Rate (ESR) D. Nausea and Vomiting

B, hypoalbumenriaThey may be seen with conditions in which the body does not properly absorb and digest protein, such as Crohn's disease or celiac disease, or in which large volumes of protein are lost from the intestines.

The nurse is performing an abdominal assessment for a patient with diarrhea and auscultates a loud rumbling sound in the left lower quadrant. What will the nurse document this sound as on the nurses notes? A. Loud bowel sounds B. Borborygmus C. Tenemus D. peristalsis

B. Borborygmus

Which of the following foods should a nurse encourage for a client who is experiencing constipation?

B. Fresh fruit and whole wheat toast A high-fiber diet promotes normal bowel elimination. The choice of fruit and toast is the highest fiber option. Macaroni and cheese, beef tips and noodles, and mashed potatoes and gravy are lower-fiber options. Besides, ain't no body got time to make mac-n-cheese, fancy beef tips with noodles, and mashed potatoes and gravy. If you do, please share with Jan.

A client with inflammatory bowel disease is receiving TPN (total parenteral nutrition) via an infusion pump. When administering TPN it is essential that the nurse: A. monitor the clients blood glucose level Q2H at the bedside with a glucometer B. change the TPN solution bag every 24 hours even if there is solution left in the bag C. instruct the client to breathe shallowly when changing the TPN tubing using sterile technique D. speed up the rate of the TPN infusion if the amount delivered has fallen behind the prescribed hourly rate

B. TPN solutions are high in glucose and are administered at room temperature, factors that increase the risk of microbial growth in the solution. They should be changed daily or sooner if they appear cloudy.

The nurse is irrigating a colostomy when the patient says, "you will have to stop, I am cramping so badly." What is the priority action by the nurse? A. Inform the patient that it will only last a minute and continue with the procedure B. clamp the tubing and give the patient a rest period C. Stop the irrigation and remove the tube D. replace the fluid with cooler water since it is probably too warm

B. clamp the tubing and give the patient a rest period

The nurse is caring for a patient who has malabsorption syndrome with an undetermined cause. What procedure will the nurse assist with that is the best diagnostic test for this illness? A. Ultrasound B. endoscopy with mucosal biopsy C. stool specimen for ova and parasites D. pancreatic function tests

B. endoscopy with mucosal biopsy

The nurse is assessing a patient with appendicitis. The nurse is attempting to elicit a Rovsing sign. Where should the nurse palpate for this indicator of acute appendicitis? A. Right lower quadrant B. left lower quadrant C. right upper quadrant D. left upper quadrant

B. left lower quadrant

The nurse is admitting a patient with a diagnosis of diverticulitis and assesses that the patient has a board like abdomen, no bowel sounds, an reports severe abdominal pain. What is the nurses first action? A. Start an IV with lactated ringer solution B. notify the health care provider C. administer a retention enema D. administer in opioid analgesic

B. notify the health care provider

The nurse is assigned to care for a patient two days after an appendectomy due to a ruptured appendix with resultant peritonitis. The nurse has just assisted the patient with ambulation to the bedside commode when the patient points to the surgical site and informs the nurse that "something gave way." What does the nurse suspect may have occurred? A. A drain may have become dislodged B. wound dehiscence has occurred C. Infection has developed D. the surgical wound has begun to bleed

B. wound dehiscence has occurred

Which of the following would be the highest priority information to include in preoperative teaching for a 68-year-old patient scheduled for a colectomy? "A: How to care for the wound B: How to deep breathe and cough C: The location and care of drains after surgery D: What medications will be used during surgery"

B: How to deep breathe and coughBecause anesthesia, an abdominal incision, and pain can impair the patient's respiratory status in the postoperative period, it is of high priority to teach the patient to cough and deep breathe. Otherwise, the patient could develop atelectasis and pneumonia, which would delay early recovery from surgery and hospital discharge.

"Which of the following would be the highest priority information to include in preoperative teaching for a 68-year old patient scheduled for a colectomy? "A. how to care for the wound (B). how to deep breath and cough C. the location and care of drains after surgery D. what medications will be used during surgery"

Because anasthesia, an abdominal incision, and pain can impair the patient's respiratory status in the postoperative period, it is of high priority to teach the pt. to cough and deep breathe. Otherwise, the pt. could develop atelectasis and pneumonia, which would delay early recovery from surgery and hospital discharge.

The nurse is questioning a client with Crohn disease about the presence of current symptoms. Which symptom should the nurse expect to find in the​ client? (Select all that​ apply.) Left lower quadrant abdominal pain relieved by defecation Blood in the stool ​Fissures, ulcers,​ fistulas, and abscesses of the anorectal area Persistent diarrhea Malaise

Blood in the stool ​Fissures, ulcers,​ fistulas, and abscesses of the anorectal area Persistent diarrhea Malaise Blood may or may not be present in the stool of a client with Crohn​ disease, and the nurse would need to assess for it. A client with Crohn disease will have​ right, not​ left, lower quadrant abdominal pain relieved by defecation. Persistent diarrhea is a symptom of Crohn disease. Malaise may be present in a client with Crohn disease.​ Fissures, ulcers,​ fistulas, and abscesses of the anorectal area may be present in a client with Crohn​ disease, and the nurse would assess the client for them.

A client who has a history of chronic ulcertaive colitis is diagnosed with anemia. The nurse intreprets that which factor is most likely responxible for the anemia? a. Blood Loss b. Intestinal hookworm c. intestinal malaborption d. Decreased intake of dietary iron

Blood loss, The client with chronic ulcerative colitis is most likely enemic as a result of chronic blood loss in small amounts tha occurs with exacerbations of the disease. These clients often have bloody stools and are at increasd risk for anemia. There is no information in the question to supprot options b. or d. In ulcerative colitis, the large intestine is involves, not the small intestine, where vitamin B12 and folic acid are absorbed (option c.)

A patient is receiving treatment for ulcerative colitis by taking Azathioprine. Which physician's order would the nurse question if received?* A. Ambulate the patient twice day B. Low-fiber and high-protein diet C. Administer varicella vaccine intramuscularly D. Administer calcium carbonate by mouth daily

C

A patient with Crohn's Disease is most likely to have the disease is what part of the GI tract? A. Rectum B. Duodenum of the small intestine C. Terminal Ileum D. Descending colon

C

A physician has prescribed a patient with a severe case of Crohn's Disease to take a drug that works by suppressing the immune system. This medication achieves this by blocking a protein that plays a role the inflammatory process. Which drug does this describe? A. Azathioprine B. Sulfasalazine C. Infliximab D. Prednisone

C

The PACU has received a semiconscious patient from the operating room and reviews the chart for orders related to positioning of the patient. There are no specific orders on the chart related to specific orders for the patient's position. In this situation, in what position will the nurse place the patient? A. Trendelenburg position B. Prone position C. Side-lying position D. Supine position

C) If the patient is not fully conscious, place the patient in the side-lying position, unless there is an ordered position on the patient's chart.

an 18 yr old is admitted with an acute onset of right lower quadrant pain. Appendicitis is suspected. For which clinical indicator should the nurse assess the client to determine if the pain is secondary to appendicitis A) urinary retention B) gastric hyperacidity C) rebound tenderness D) increased lower bowel motility

C) rebound tenderness is a classic subjective sign of appendicitis

A client who has had ulcerative colitis for the past 5 years is admitted to the hospital with an exacerbation of the disease. Which of the following factors was most likely of greatest significance in causing an exacerbation of ulcerative colitis? 1. A demanding and stressful job. 2. Changing to a modified vegetarian diet 3. Beginning a weight-training program 4. Walking 2 miles every day

C, Stressful and emotional events have been clearly linked to exacerbations of ulcerative colitis, although their role in the etiology of the disease has been disproved. A modified vegetarian diet or an exercise program is an unlikely cause of the exacerbation

The client with inflammatory bowel disease had surgery to create a continent kock's ileostomy yesterday. The client refuses to look at the stoma when the nurse is teaching stoma care. The best nursing intervention for the nurse to take is to: "A. notify the physician that the patient is depressed. B. continue patient teaching to meet care goals. C. encourage the client to verbalize feelings related to the stoma. D. offer the client a mirror so they can examine the stoma later when they wish to do so."

C, encourage to talk about feelingsRationale: Clients who have a stoma often experience alterations in body image. The nurse should encourage the client to verbalize feelings related to disease process and stoma

Following bowel resection, a patient has a nasogastric tube to suction, but complains of nausea and abdominal distention. The nurse irrigates the tube prn as ordered, but the irrigating fluid does not return. Which of the following should be the priority action by the nurse? A. Notify the physician B. Auscultate for bowel sounds. C. Reposition the tube and check for placement. D. Remove the tube and replace it with a new one.

C, reposition the tube and check for placementThe tube may be resting against the stomach wall. The first action by the nurse, since this was intestinal surgery (not gastric surgery), is to reposition the tube and check it again for placement.

Patients may experience which problem 24-48 hrs post-op as a result of anesthetics? A. colitis B. Stomatitis C. Paralytic ileus D. Gastrocolic reflux

C. After surgery, pts are clients are at risk for paralytic ileus as a result of anesthesia.

What would be the most effective way for a nurse to validate "informed consent"? A. Ask the family whether the patient understands the procedure. B. Check the chart for a completed and signed consent form. C. Ask the patient what he or she understands regarding the procedure. D. Determine from the physician what was discussed with the patient.

C. Ask the patient what he or she understands regarding the procedure. Informed consent in the health care setting is a process whereby a patient is informed of the risks, benefits, and alternatives of a certain procedure, and then gives consent for it to be done. The piece of paper is simply evidence that the informed consent process has been done.

Nurse is caring for a patient with a diagnosis of ulcerative colitis. Which finding, if noted on assessment of the client, would the nurse report to the Dr? A. Hypotension B. Bloody diarrhea C. Rebound tenderness D. Hemoglobin of 12 mg/ dl

C. Rebound tenderness because this could indicate peritonitis.

A client has surgery for a perforated appendix with localized peritonis. In which position should the nurse place the client? A) Sims position B) trendelenburg C) semi-fowlers D)dorsal recumbant

C. Semi-fowlers aids in drainage and prevents spread of infection throughout the abodominal cavity.

A patient arrives in the emergency department reporting right lower abdominal pain that began 4 hours ago and is getting worse. The nurse assesses rebound tenderness at mcburney point. What does this assessment data indicate to the nurse A. Crohn's disease B. ulcerative colitis C. appendicitis D. diverticulitis

C. appendicitis

A patient is admitted to the hospital after not having had a bowel movement in several days. The nurse observes the patient is having small liquid stools, a grossly distended abdomen, and abdominal cramping. What complication can this patient develop related to this problem? A. Appendicitis B. rectal fissures C. bowel perforation D. diverticulitis

C. bowel perforation

In the absence of postoperative vomiting, GI suctioning, and wound drainage, the physiologic responses to the stress of surgery are most likely to cause A. diuresis. B. hyperkalemia. C. fluid overload. D. impaired blood coagulation.

C. fluid overload. Secretion and release of aldosterone and cortisol from the adrenal gland and ADH from the posterior pituitary as a result of the stress response cause fluid retention during the first 2 to 5 days postoperatively, and fluid overload is possible during this time. Aldosterone causes renal potassium loss with possible hypokalemia, and blood coagulation is enhanced by cortisol.

The nurse is caring for a patient who has had an appendectomy. What is the best position for the nurse to maintain the patient in after the surgery? A. Prone B. Sims left lateral C. high Fowler D. supine with head of bed elevated 15 degrees

C. high Fowler

A patient is not having daily bowel movements and has begun taking a laxative for this problem. What should the nurse educate the patient about regarding laxative use? A. When taking the laxatives, plenty of fluids should be taken as well B. the laxatives should be taken no more than three times a week or laxative addiction will result C. laxatives should not be routinely taken due to destruction of nerve endings in the colon D. laxatives should never be the first response for the treatment of Constipation; Natural methods should be employed first

C. laxatives should not be routinely taken due to destruction of nerve endings in the colon

A patient is being seen in the clinic reporting painful hemorrhoids. The nurse assesses the patient and observes the hemorrhoids are prolapsed but able to be placed back in the rectum manually. the nurse documents the hemorrhoids as what degree? A. First degree B. second degree C. third degree D. 4th degree

C. third degree

A client with Crohn's disease is admitted to the hospital with a history of chronic, bloody diarrhea, weight loss, and signs of general malnutrition. The client has anemia, a low serum albumin level, and signs of negative nitrogen balance. The nurse concludes that the client's health status is related to a major deficiency of: 1. Iron 2. Protein 3. Vitamin C 4. Linoleic acid

CORRECT ANSWER 2: Protein deficiency causes a low serum albumin level, which permits fluid shifts from the intravascular to the interstitial compartment, resulting in edema. Decreased protein also causes anemia; protein intake must be increased. Although a deficiency of iron will result in anemia, it will not cause the other adaptations. Vitamin C and linoleic acid are unrelated to these adaptations.

The nurse is monitoring a client diagnosed with appendicitis who is scheduled for surgery in 2 hours. The client begins to complain of increased abdominal pain and begns to vomit. On assessment, the nurse notes that the abdomen is distended and bowel sounds are diminished. Which is the appropriate nursing intervention? "1. Notify the Physician 2. Administer the prescribed pain medication 3. Call and ask the operating room team to perform the surgery as soon as possible 4. Reposition the client and apply a heating pad on warm setting to the client's abdomen

CORRECT ANSWER: 1" "1. Based on the assessment information the nurse should suspect peritonitis, a complication that is associated with appendicitis, and notify the physician. 2. Administering pain medication is not an appropriate intervention 3. Scheduling surgical time is not within the scope of practice of an RN. 4. Heat should never be applied to the abdomen of a patient suspected of having peritonitis because of the risk of rupture."

In planning the care for the patient with Crohn's disease, the nurse recognizes that a major difference between ulcerative colitis and Crohn's disease is that Crohn's disease: a) frequently results in toxic megacolon b) causes fewer nutritional deficiencies than does ulcerative colitis. c) often recurs after surgery, whereas ulcerative colitis is curable with a colectomy. d) is manifested by rectal bleeding and anemia more frequently than is ulcerative colitis.

CORRECT ANSWER: CBecause there is a high recurrence rate after surgical treatment of Crohn's disease, medications are the preferred treatment.

The nurse prepares teaching for the parents of a child with inflammatory bowel disease​ (IBD). Which mineral should the nurse encourage the parents to provide to the child to prevent future​ complications? Magnesium Sodium Calcium Potassium

Calcium Children with IBD are at higher risk for poor bone density as they get older. It is important that they take in an adequate amount of calcium to promote bone health and strength.​ Magnesium, sodium, and potassium do not help to maintain strong bones.

Clostridium difficile

Contact: 1) by factors that cause an overgrowth of C. difficile. 2) by contact with the organism Symptoms: Chapter 46; Page 1092

The nurse notes that a client with inflammatory bowel disease​ (IBD) has a stoma with a nipple valve. Which type of ostomy should the nurse document that this client​ has? Ileostomy Temporary ostomy Continent ileostomy Loop ostomy

Continent ileostomy A continent ileostomy is a procedure where an​ intra-abdominal reservoir is created and a nipple valve is attached to prevent drainage of stool from the reservoir. A nipple valve is not used during an​ ileostomy, loop​ ostomy, or temporary ostomy.

The client with sever abdominal pain is being evaluated for appendicitis. What is the most common cause of appendicistis? http://nursing.slcc.edu/nclexrn3500/ 1. Rupture of the appendix 2.Obstruction of the appendix 3 A high-fat diet 4. A duodenal ulcer

Correct 2 Appendicitis most commonly results from obstruction of the appendix, which may lead to rupture. A high-fat diet or duodenal ulcer doesn't cause appendicitis; however, a client may require dietary restrictions after an appendectomy

which statement made by the client who is postoperative abdominal surgery indicates the discharge teaching has been effective? 1. "i will take my temp each week and report any elevation." 2. "i will not need any pain meds when i go home." 3. i will take all of my antibiotics until they are gone." 4. i will not take a shower until my three month check up.

Correct 3 1. the client should check the temp twice a day. 2. it is not realistic to expect the client to experience no pain after surgery. 3 (CORRECT): this statement about taking all the antibiotics ordered indicates the teaching is effective. 4. clients may shower after surgery, but not taking a tub bath for three months after surgery is too long a time.

A client complains of severe pain in the right lower quadrant of the abdomen. To assist with pain relief, the nurse should take which of the following actions? "1. Encourage the client to change positions frequently in bed 2. Massage the right lower quadrant fo the abdomen 3. Apply warmth to the abdomen with a heating pad 4. Use comfort measures and pillows to position the client"

Correct 4 "1. ""Encourage the client..."" - unnecesary movement will increase pain and should be avoided 2. ""Massage the lower..."" - if appendicitis is suspected, massorge or palpation should never be performed as thes actions may cause the appendix to rupture 3. ""Apply warmth..."" - if pain is casused by appendicitis, increased circulation from the heat may cause appendix to rupture 4. ""Use comfort measures..."" - CORRECT: non-pharmacological methods of pain relief"

"A client with acute appendicitis develops a fever, tachycardia, and hypotension. Based on these assessment findings, the nurse should further assess the client for which of the following complications?... "1. Deficient fluid volume. 2. Intestinal obstruction. 3. Bowel ischemia. 4. Peritonitis

Correct 4 "Complications of acute appendicitis are perforation, peritonitis, and abscess development. Signs of the development of peritonitis include abdominal pain and distention, tachycardia, tachypnea, nausea, vomiting, and fever. Because peritonitis can cause hypovolemic shock, hypotension can develop. Deficient fluid volume would not cause a fever. Intestinal obstruction would cause abdominal distention, diminished or absent bowel sounds, and abdominal pain. Bowel ischemia has signs and symptoms similar to those found with intestinal obstruction."

Which of the following would confirm a diagnosis of appendicitis? "a. The pain is localized at a position halfway between the umbilicus and the right iliac crest. b. Mr. Liu describes the pain as occurring 2 hours after eating c. The pain subsides after eating d. The pain is in the left lower quadrant"

Correct A "Pain over McBurney's point, the point halfway between the umbilicus and the iliac crest, is diagnosis for appendicitis. Options b and c are common with ulcers; option d may suggest ulcerative"

Which diagnostic test is used first to evaluate a client with upper GI bleeding? "a) Hemoglobin levels and hematocrit (HCT) b) Endoscopy c) Arteriography d) Upper GI series

Correct Answer: (A) Hgb and Hct levels" Hemoglobin and HCT are typically performed first in clients with upper GI bleeding to evaluate the extent of blood loss. Endoscopy is then performed to directly visualize the upper GI tract and locate the source of bleeding. An upper GI series, or barium study, usually isn't the diagnostic method of choice, especially in a client with acute active bleeding who's vomiting and unstable. An upper GI series is also less accurate than endoscopy. Although an upper GI series might confirm the presence of a lesion, it wouldn't necessarily reveal whether the lesion is bleeding. Arteriography is an invasive study associated with life-threatening complications and wouldn't be used for an initial evaluation.

"When evaluating a male client for complications of acute pancreatitis, the nurse would observe for: "a. increased intracranial pressure. b. decreased urine output. c. bradycardia. d. hypertension."

Correct Answer: B Rationale: Acute pancreatitis can cause decreased urine output, which results from the renal failure that sometimes accompanies this condition. Intracranial pressure neither increases nor decreases in a client with pancreatitis. Tachycardia, not bradycardia, usually is associated with pulmonary or hypovolemic complications of pancreatitis. Hypotension can be caused by a hypovolemic complication, but hypertension usually isn't related to acute pancreatitis."

Which of the following position should the client with appendicitis assume to relieve pain ? A. Prone B. Sitting C. Supine D. Lying with legs drawn up

Correct Answer: D Lying still with legs drawn up towards chest helps relive tension on the abdominal muscle, which helps to reduce the amount of discomfort felt. Lying flat or sitting may increase the amount of pain experienced

A nurse is making a home health visit and finds the client experiencing right lower quadrant abdominal pain, which has decreased in intensity over the last day. The client also has a rigid abdomen and a temperature of 103.6 F. The nurse should intervene by: a) administer Tylenol (acetaminophen) for the elevated temperature b) advising the client to increase oral fluids c) asking the client when she last had a bowel movement d) notifying the physician

Correct D D. The client symptoms indicate appendicitis which requires immediate attention

"During the assessment of a patient with acute abdominal pain, the nurse should: a. perform deep palpation before auscultation b. obtain blood pressure and pulse rate to determine hypervolemic changes c. auscultate bowel sounds because hyperactive bowel sounds suggest paralytic ileus d. measure body temperature because an elevated temperature may indicate an inflammatory or infectious process.

Correct D Rationale: for the patient complaining of acute abdominal pain, nurse should take vital signs immediately. Increased pulse and decreasing blood pressure are indicative of hypovolemia. An elevated temperature suggests an inflammatory infectious process. Intake and output measurements provide essential information about the adequate of vascular volume. Inspect abdomen first and then auscultate bowel sounds. Palpation is performed next and should be gentle.

Which of the following would indicate that Bobby's appendix has ruptured? " a) diaphoresis b) anorexia c) pain at Mc Burney's point d) relief from pain

Correct D all are normal signs of having appendicits and once you have relief from pain means you could have a rupture.

The client is diagnosed with Crohn's disease, also known as regional enteritis. Which statement by the client would support this diagnosis? 1. "My pain goes away when I have a bowel movement." 2."I have bright red blood in my stool all the time." 3."I have episodes of diarrhea and constipation." 4."My abdomen is hard and rigid and I have a fever."

Correct answer: #1 pain goes away w/ BM, The terminal ileum is the most common site for regional enteritis and causes right lower quadrant pain that is relieved by defecation.

The nurse would increase the comfort of the patient with appendicitis by: "a. Having the patient lie prone b. Flexing the patient's right knee c. Sitting the patient upright in a chair d. Turning the patient onto his or her left side

Correct answer: B" The patient with appendicitis usually prefers to lie still, often with the right leg flexed to decrease pain.

What should the nurse instruct the patient to do to best enhance the effectiveness of a daily dose of docusate sodium (Colace)? a. Take a dose of mineral oil at the same time. b. Add extra salt to food on at least one meal tray. c. Ensure dietary intake of 10 g of fiber each day. d. Take each dose with a full glass of water or other liquid.

Correct answer: D Rationale: Docusate lowers the surface tension of stool, permitting water and fats to penetrate and soften the stool for easier passage. The patient should take the dose with a full glass of water and should increase overall fluid intake, if able, to enhance effectiveness of the medication. Dietary fiber intake should be a minimum of 20 g daily to prevent constipation. Mineral oil and extra salt are not recommended.

The appropriate collaborative therapy for the patient with acute diarrhea caused by a viral infection is to a. increase fluid intake. b. administer an antibiotic. c. administer antimotility drugs. d. quarantine the patient to prevent spread of the virus.

Correct answer: a Rationale: Acute diarrhea resulting from infectious causes (e.g., virus) is usually self-limiting. The major concerns are transmission prevention, fluid and electrolyte replacement, and resolution of the diarrhea. Antidiarrheal agents are contraindicated in the treatment of infectious diarrhea because they potentially prolong exposure to the infectious organism. Antibiotics are rarely used to treat acute diarrhea. To prevent transmission of diarrhea caused by a virus, hands should be washed before and after contact with the patient and when body fluids of any kind are handled. Vomitus and stool should be flushed down the toilet, and contaminated clothing should be washed immediately with soap and hot water.

The nurse is conducting discharge teaching for a patient with metastatic lung cancer who was admitted with a bowel impaction. Which instructions would be most helpful to prevent further episodes of constipation? a. Maintain a high intake of fluid and fiber in the diet. b. Reduce intake of medications causing constipation. c. Eat several small meals per day to maintain bowel motility. d. Sit upright during meals to increase bowel motility by gravity.

Correct answer: a Rationale: Increased fluid intake and a high-fiber diet reduce the incidence of constipation caused by immobility, medications, and other factors. Fluid and fiber provide bulk that in turn increases peristalsis and bowel motility. Analgesics taken for lung cancer probably cannot be reduced. Other medications may decrease constipation, but it is best to avoid laxatives. Eating several small meals per day and position do not facilitate bowel motility. Defecation is easiest when the person sits on the commode with the knees higher than the hips.

A stroke patient who primarily uses a wheelchair for mobility has diarrhea with fecal incontinence. What should the nurse assess first? a. Fecal impaction b. Perineal hygiene c. Dietary fiber intake d. Antidiarrheal agent use

Correct answer: a Rationale: Patients with limited mobility are at risk for fecal impactions due to constipation that may lead to liquid stool leaking around the hardened impacted feces, so assessing for fecal impaction is the priority. Perineal hygiene can be assessed at the same time. Assessing the dietary fiber and fluid intake and antidiarrheal agent use will be assessed and considered next.

The nurse is preparing to insert a nasogastric (NG) tube into a 68-year-old female patient who is nauseated and vomiting. She has an abdominal mass and suspected small intestinal obstruction. The patient asks the nurse why this procedure is necessary. What response by the nurse is most appropriate? a. "The tube will help to drain the stomach contents and prevent further vomiting." b. "The tube will push past the area that is blocked and thus help to stop the vomiting." c. "The tube is just a standard procedure before many types of surgery to the abdomen." d. "The tube will let us measure your stomach contents so that we can plan what type of IV fluid replacement would be best."

Correct answer: a Rationale: The NG tube is used to decompress the stomach by draining stomach contents and thereby prevent further vomiting. The NG tube will not push past the blocked area. Potential surgery is not currently indicated. The location of the obstruction will determine the type of fluid to use, not measure the amount of stomach contents.

Two days following a colectomy for an abdominal mass, a patient reports gas pains and abdominal distention. The nurse plans care for the patient based on the knowledge that the symptoms are occurring as a result of a. impaired peristalsis. b. irritation of the bowel. c. nasogastric suctioning. d. inflammation of the incision site.

Correct answer: a Rationale: Until peristalsis returns to normal following anesthesia, the patient may experience slowed gastrointestinal motility leading to gas pains and abdominal distention. Irritation of the bowel, nasogastric suctioning, and inflammation of the surgical site do not cause gas pains or abdominal distention.

Assessment findings suggestive of peritonitis include a. rebound abdominal pain. b. a soft, distended abdomen. c. dull, continuous abdominal pain. d. observing that the patient is restless.

Correct answer: a Rationale: With peritoneal irritation, the abdomen is hard, like a board, and the patient has severe abdominal pain that is worse with any sudden movement. The patient lies very still. Palpating the abdomen and releasing the hands suddenly causes sudden movement within the abdomen and severe pain. This is called rebound tenderness.

The nurse is planning care for a 68-year-old patient with an abdominal mass and suspected bowel obstruction. Which factor in the patient's history increases the patient's risk for colorectal cancer? a. Osteoarthritis b. History of colorectal polyps c. History of lactose intolerance d. Use of herbs as dietary supplements

Correct answer: b Rationale: A history of colorectal polyps places this patient at risk for colorectal cancer. *This tissue can degenerate over time and become malignant*. Osteoarthritis, lactose intolerance, and the use of herbs do not pose additional risk to the patient.

What information would have the highest priority to be included in preoperative teaching for a 68-year-old patient scheduled for a colectomy? a. How to care for the wound b. How to deep breathe and cough c. The location and care of drains after surgery d. Which medications will be used during surgery

Correct answer: b Rationale: Because anesthesia, an abdominal incision, and pain can impair the patient's respiratory status in the postoperative period, it is of high priority to teach the patient to cough and deep breathe. Otherwise, the patient could develop atelectasis and pneumonia, which would delay early recovery from surgery and hospital discharge. Care for the wound and location and care of the drains will be briefly discussed preoperatively, but done again with higher priority after surgery. Knowing which drugs will be used during surgery may not be meaningful to the patient and should be reviewed with the patient by the anesthesiologist.

A patient who is given a bisacodyl (Dulcolax) suppository asks the nurse how long it will take to work. The nurse replies that the patient will probably need to use the bedpan or commode within which time frame after administration? a. 2-5 minutes b. 15-60 minutes Correct c. 2-4 hours Incorrect d. 6-8 hours

Correct answer: b Rationale: Bisacodyl suppositories usually are effective within 15 to 60 minutes of administration, so the nurse should plan accordingly to assist the patient to use the bedpan or commode.

The nurse should administer an as-needed dose of magnesium hydroxide (MOM) after noting what information while reviewing a patient's medical record? a. Abdominal pain and bloating b. No bowel movement for 3 days c. A decrease in appetite by 50% over 24 hours d. Muscle tremors and other signs of hypomagnesemia

Correct answer: b Rationale: MOM is an osmotic laxative that produces a soft, semisolid stool usually within 15 minutes to 3 hours. This medication would benefit the patient who has not had a bowel movement for 3 days. MOM would not be given for abdominal pain and bloating, decreased appetite, or signs of hypomagnesemia.

A colectomy is scheduled for a 38-year-old woman with ulcerative colitis. The nurse should plan to include what prescribed measure in the preoperative preparation of this patient? a. Instruction on irrigating a colostomy b. Administration of a cleansing enema c. A high-fiber diet the day before surgery d. Administration of IV antibiotics for bowel preparation

Correct answer: b Rationale: Preoperative preparation for bowel surgery typically includes *bowel cleansing with antibiotics, such as oral neomycin and cleansing enemas, including Fleet enemas*. Instructions to irrigate the colostomy will be done postoperatively. Oral antibiotics are given preoperatively, and an IV antibiotic may be used in the OR. A clear liquid diet will be used the day before surgery with the bowel cleansing.

A patient with stage I colorectal cancer is scheduled for surgery. Patient teaching for this patient would include an explanation that a. chemotherapy will begin after the patient recovers from the surgery. b. both chemotherapy and radiation can be used as palliative treatments. c. follow-up colonoscopies will be needed to ensure that the cancer does not recur. d. a wound, ostomy, and continence nurse will visit the patient to identify an abdominal site for the ostomy.

Correct answer: c Rationale: Stage 1 colorectal cancer is treated with surgical removal of the tumor and reanastomosis, and so there is no ostomy. Chemotherapy is not recommended for stage I tumors. Follow-up colonoscopy is recommended because colorectal cancer can recur.

In planning care for the patient with Crohn's disease, the nurse recognizes that a major difference between ulcerative colitis and Crohn's disease is that Crohn's disease a. frequently results in toxic megacolon. b. causes fewer nutritional deficiencies than ulcerative colitis. c. often recurs after surgery, whereas ulcerative colitis is curable with a colectomy. d. is manifested by rectal bleeding and anemia more frequently than is ulcerative colitis.

Correct answer: c Rationale: Ulcerative colitis affects only the colon and rectum; it can cause megacolon and rectal bleeding, but not nutrient malabsorption. Surgical removal of the colon and rectum cures it. Crohn's disease usually involves the ileum, where bile salts and vitamin cobalamin are absorbed. After surgical treatment, disease recurrence at the site is common.

The nurse is preparing to administer a dose of bisacodyl (Dulcolax). In explaining the medication to the patient, the nurse would explain that it acts in what way? a. Increases bulk in the stool b.Lubricates the intestinal tract to soften feces c. Increases fluid retention in the intestinal tract d. Increases peristalsis by stimulating nerves in the colon wall

Correct answer: d Rationale: Bisacodyl is a stimulant laxative that aids in producing a bowel movement by irritating the colon wall and stimulating enteric nerves. It is available in oral and suppository forms. Fiber and bulk forming drugs increase bulk in the stool; water and stool softeners soften feces, and saline and osmotic solutions cause fluid retention in the intestinal tract.

The nurse is caring for a 68-year-old patient admitted with abdominal pain, nausea, and vomiting. The patient has an abdominal mass, and a bowel obstruction is suspected. The nurse auscultating the abdomen listens for which type of bowel sounds that are consistent with the patient's clinical picture? a. Low-pitched and rumbling above the area of obstruction b. High-pitched and hypoactive below the area of obstruction c. Low-pitched and hyperactive below the area of obstruction d. High-pitched and hyperactive above the area of obstruction

Correct answer: d Rationale: Early in intestinal obstruction, the patient's bowel sounds are hyperactive and high-pitched, sometimes referred to as "tinkling" above the level of the obstruction. *This occurs because peristaltic action increases to "push past" the area of obstruction*. As the obstruction becomes complete, bowel sounds decrease and finally become absent.

The nurse would question the use of which cathartic agent in a patient with renal insufficiency? a. Bisacodyl (Dulcolax) b. Lubiprostone (Amitiza) c. Cascara sagrada (Senekot) d. Magnesium hydroxide (Milk of Magnesia)

Correct answer: d Rationale: Milk of Magnesia may cause hypermagnesemia in patients with renal insufficiency. The nurse should question this order with the health care provider. Bisacodyl, lubiprostone, and cascara sagrada are safe to use in patients with renal insufficiency as long as the patient is not currently dehydrated.

The nurse is preparing to administer a scheduled dose of docusate sodium (Colace) when the patient reports an episode of loose stool and does not want to take the medication. What is the appropriate action by the nurse? a. Write an incident report about this untoward event. b. Attempt to have the family convince the patient to take the ordered dose. c. Withhold the medication at this time and try to administer it later in the day. d. Chart the dose as not given on the medical record and explain in the nursing progress notes.

Correct answer: d Rationale: Whenever a patient refuses medication, the dose should be charted as not given with an explanation of the reason documented in the nursing progress notes. In this instance, the refusal indicates good judgment by the patient, and the patient should not be encouraged to take it today.

"A client is admitted with a diagnosis of acute appendicitis. When assessing the abdomen, the nurse would expect to find rebound tenderness at which location? a) Left lower quadrant b) Left upper quadrant c) Right upper quadrant d) Right lower quadrant

Correct answer: d) Right lower quadrant" Rationale: The pain of acute appendicitis localizes in the right lower quadrant (RLQ) at McBurney's point, an area midway between the umbilicus and the right iliac crest. Often, the pain is worse when manual pressure near the region is suddenly released, a condition called rebound tenderness.

The nurse performs a detailed assessment of the abdomen of a patient with a possible bowel obstruction, knowing that manifestations of an obstruction in the large intestine are (select all that apply) a. persistent abdominal pain. b. marked abdominal distention. c. diarrhea that is loose or liquid. d. colicky, severe, intermittent pain. e. profuse vomiting that relieves abdominal pain.

Correct answers: a, b Rationale: With lower intestinal obstructions, abdominal distention is markedly increased and pain is persistent. Onset of a large intestine obstruction is gradual, vomiting is rare, and there is usually absolute constipation, not diarrhea.

When a 35-year-old female patient is admitted to the emergency department with acute abdominal pain, which possible diagnosis should you consider that may be the cause of her pain (select all that apply)? a. Gastroenteritis b. Ectopic pregnancy c. Gastrointestinal bleeding d. Irritable bowel syndrome e. Inflammatory bowel disease

Correct answers: a, b, c, d, e Rationale: All these conditions could cause acute abdominal pain.

Which clinical manifestations of inflammatory bowel disease are common to both patients with ulcerative colitis (UC) and Crohn's disease (select all that apply)? a. Restricted to rectum b. Strictures are common. c. Bloody, diarrhea stools d. Cramping abdominal pain e. Lesions penetrate intestine.

Correct answers: c, d Rationale: Clinical manifestations of UC and Crohn's disease include bloody diarrhea, cramping abdominal pain, and nutritional disorders. Intestinal lesions associated with UC are usually restricted to the rectum before moving into the colon. Lesions that penetrate the intestine or cause strictures are characteristic of Crohn's disease.

"The nurse is admitting a client with acute appendicitis to the emergency department. The client has abdominal pain of 10 on a pain scale of 1 to 10. The client will be going to surgery as soon as possible. The nurse should: "1. Contact the surgeon to request an order for a narcotic for the pain. 2. Maintain the client in a recumbent position. 3. Place the client on nothing-by-mouth (NPO) status. 4. Apply heat to the abdomen in the area of the pain."

Correct: 3 - no rationale

"A client has an appendectomy and develops peritonitis. The nurse should asses the client for an elevated temperature and which additional clinical indication commonly associated with peritonitis? "1. hyperactivity 2. extreme hunger 3. urinary retention 4. local muscular rigidity

Correct: 4 muscular rigidity over the affected area is a classic sign of peritonitis

Which client requires immediate nursing intervention? "The client who: a) complains of epigastric pain after eating. b) complains of anorexia and periumbilical pain. c) presents with ribbonlike stools. d) presents with a rigid, boardlike abdomen.

Correct: D A rigid, boardlike abdomen is a sign of peritonitis, a possibly life-threatening condition. Epigastric pain occurring 90 minutes to 3 hours after eating indicates a duodenal ulcer. Anorexia and periumbilical pain are characteristic of appendicitis. Risk of rupture is minimal within the first 24 hours, but increases significantly after 48 hours. A client with a large-bowel obstruction may have ribbonlike stools.

A patient diagnosed with pancolitis is experiencing extreme abdominal distension, pain 10 on 1-10 scale in the abdomen, temperature of 103.6 'F, HR 120, and profuse diarrhea. What complication due you suspect the pain is experiencing?* A. Fistulae B. Stricture C. Bowel obstruction D. Toxic megacolon

D

A patient experiencing a flare-up with Crohn's Disease is ordered complete bowel rest by the physician. You are administering TPN (total parental nutrition) per physician order. When developing the patient's nursing plan of care, which nursing diagnosis is MOST important to include in the care plan? A. Risk for allergy response B. Risk for unstable blood glucose level C. Risk for imbalance nutrition: more than body requirements D. Risk for imbalanced nutrition: less than body requirements

D

A patient is admitted with ulcerative colitis. In the physician's notes, it is stated that the patient's barium enema results showed the patient has colitis that starts in the rectum and extends into the sigmoid and descending colon. As the nurse, you know that this is what type of ulcerative colitis?* A. Right-sided colitis B. Proctosigmoiditis C. Ulcerative procotitis D. Left-sided colitis

D

You're providing teaching to a patient who has been newly diagnosed with Crohn's Disease. Which statement by the patient's spouse requires re-education? A. "Crohn's Disease can be scattered throughout the GI tract in patches with some areas appearing healthy while others are diseased." B. "There is no cure for Crohn's Disease." C. "Strictures are a common complication with Crohn's Disease." D. "Crohn's Disease can cause the haustra of the large intestine to lose its form."

D

A client with acute colcerative colitis requests a snack. Which of the following foods is the most appropriate to give the client? A. Carrots and ranch dip B. Whole grain cereal and milk C. A cup of popcorn and a cola D. Applesauce and a graham cracker

D, appelsauce and graham cracker, The diet for a client with ulcerative coliits should be a low-fiber, low residue diet. The nurse should avoid foods such as whole grains, nuts and fresh fruit or vegetables. Typically lactose containing foods are also poorly tolerated. The client should also avoid caffeine, pepper, and alcohol.

While a nurse is caring for a patient who is scheduled to have surgery in 2 hours, the patient states, "My doctor was here and told me a lot of stuff I didn't understand and then I signed a paper for her." To fulfill the role of advocate, what is the best nursing action? A. Reassure the patient that the surgery will go as planned. B. Explain the surgery and possible outcomes to the patient. C. Complete her first priority, the preoperative teaching plan. D. Call the physician to return and clarify information for the patient.

D. Call the physician to return and clarify information for the patient. Examples of nursing advocacy include questioning doctors' orders, promoting patient comfort, and supporting patient decisions regarding health care choices.

A nurse is caring for a client who will perform fecal occult blood testing at home. Which of the following should the nurse include when explaining the procedure to the client? A. Eating more protein is recommended prior to testing. B. One stool specimen is sufficient for testing. C. A red color change indicates a positive test. D. The specimen cannot be contaminated with urine.

D. The specimen cannot be contaminated with urine. For fecal occult blood testing at home, the stool specimens cannot be contaminated with water or urine; three specimens from three different bowel movements are required; some proteins such as red meat, fish, and poultry can alter the test results; and a blue color indicates a positive guaiac or presence of fecal occult blood not red. Chapter 45; Page 1046

A patient with irritable bowel syndrome has been having more frequent symptoms lately and is not sure what lifestyle changes may have occurred. What suggestion can the nurse provide to identify of trigger the symptoms? A. Document how much fluid is being taken to determine if the patient is over hydrating B. discontinue the use of any medication presently being taken to determine if medication is a trigger C. begin exercise regimen and biofeedback to determine if external stress is a trigger D. keep a one to two week symptom and food diary to identify food triggers

D. keep a one to two week symptom and food diary to identify food triggers

To prevent airway obstruction in the postoperative patient who is unconscious or semiconscious, the nurse A. encourages deep breathing. B. elevates the head of the bed. C. administers oxygen per mask. D. positions the patient in a side-lying position.

D. positions the patient in a side-lying position. An unconscious or semiconscious patient should be placed in a lateral position to protect the airway from obstruction by the tongue. Deep breathing and elevation of the head of the bed are implemented to facilitate gas exchange when the patient is responsive. Oxygen administration is often used, but the patient must first have a patent airway.

In planning care for a patient with ulcerative colitis, the nurse should anticipate which of the following diagnostic procedures? a. sigmoidodscopy b. colonoscopy, c. rectal mucosa biopsy, d. all of the above

Diagnosis of ulcerative colitis is confirmed with the use of sigmoidoscopy, colonoscopy, and rectal mucosa biopsy.

The nurse reviewed data collected during the assessment of a client with inflammatory bowel disease​ (IBD). Which nursing diagnosis should the nurse use to guide this​ client's care?​ (Select all that​ apply.) ​Injury, Risk for Diarrhea ​Confusion, Acute Fluid​ Volume: Deficient: Risk for Constipation

Diarrhea Fluid​ Volume: Deficient: Risk for Constipation The client with inflammatory bowel disease is at greatest risk for deficient fluid​ volume, diarrhea, and constipation. This client does not generally display acute confusion or risk for falls.

The nurse is reviewing the record of a client with Chron's disease. Which stool characteristic should the nurse expect to note documented in the client's record? 1. Diarrhea 2. Chronic constipation 3. Constipation alternating with diarrhea 4. Stool constantly oozing from teh rectum

Diarrhea, Crohn's disease is characterized by nonbloody diarrhea of usually not more than four to five stools daily. Over time, the diarrhea episodes increase in frequency, duration, and severity. Options 2, 3, and 4 are not characteristics of Crohn's disease.

The nurse is reviewing the record of a client with Crohn's disease. Which stool characteristic should the nurse expect to note documented in the client's record? 1. Diarrhea 2. Chronic constipation 3. Constipation alternating with diarrhea 4. Stool constantly oozing from the rectum

Diarrhea, Rationale: Crohn's disease is characterized by nonbloody diarrhea of usually not more than four to five stools daily. Over time, the diarrhea episodes increase in frequency, duration and severity. Options 2, 3 and 4 are not characteristics of Crohn's diease.

What is the last resort in the management of severe constipation and practiced when all other methods such as enemas failed?

Digital Removal. Use of fingers to aid in the removal of stool from the anus. CAUTION: Stimulation of the vagus nerve can result in lowering heart rate or rhythm change. Stop procedure when this occurs. If you got this answer right, your patients are in good hands...err...fingers.

Hemorrhoids

Dilated, engorged veins in the lining of the rectum. Chapter 46; Page 1092

Cathartics

Drugs that act to promote bowel evacuation. Soften the stool and promote peristalsis. Chapter 46; Page 1091

What does it mean when an order states, "enemas until clear" or "until clear" mean?

Enemas is repeated until the patient passes fluid that is clear and contains no fecal material. Chapter 46; Page 1107

Valsalva Maneuver

Exerts pressure to expel feces through a voluntary contraction of the abdominal muscles while maintaining forced expiration against a closed airway. Chapter 46; Page 1089

True or False You can take Toradol IV for as long as you need to manage pain for IBD?

FALSE Can only give for 72hrs b/c it causes renal failure then give Tylenol

True or False: NSAIDs are used as first-line treatment for pain relief with patients with ulcerative colitis.* True False

False

True or False: Ulcerative colitis is a type of inflammatory bowel disease that causes inflammation and ulcer formation in the inner lining of the small intestine, specifically the terminal ileum.* True False

False

Incontinence

Fecal incontinence is the inability to control passage of feces and gas from the anus. Chapter 46; Page 1092

Fecal Occult Blood Test (FOBT)

Guaiac test; measures microscopic amounts of blood in feces. Chapter 46; Page 1099

Why use tap-water for enema?

HYPOtonic and exerts an osmotic pressure lower than fluid in interstitial space. Stimulates defecation before large amounts of water leave the bowel. DO NOT repeat tap-water enemas because water toxicity or circulatory overload develops if they body absorbs large amounts of water (Page 1107) AND it is a hypotonic solution, thus it pulls electrolytes from the body into the fluid. This increases the risk of electrolyte imbalance (Lecture).

What are high or low cleansing enema?

High enemas cleanse the entire colon. Low enemas cleanse ONLY rectum and sigmoid colon. Chapter 46; Page 1107

"Older patients with longstanding or severe Crohn's disease can exhibit which of the following? a Hyperalbuminemia b)Hypoalbuminemia c) Decreased sedimentation rate d)Nausea and vomiting

Hypoalbuminemia, Rationale wasn't given for this question but Crohns affects the GI tract which will affect the ability to absorb protein so it is lost through the urine.

A client who has a history of Crohn's disease is admitted to the hospital with fever, diarrhea, cramping, abdominal pain, and weight loss. The nurse should monitor the client for: ) 1. Hyperalbuminemia. 2. Thrombocytopenia. (3.) Hypokalemia. 4. Hypercalcemia.

Hypokalemia is the most expected laboratory finding owing to the diarrhea. Hypoalbuminemia can also occur in Crohn's disease; however, the client's potassium level is of greater importance at this time because a low potassium level can cause cardiac arrest. Anemia is an expected development, but thrombocytopenia is not. Calcium levels are not affected.

A nurse is caring for a client who has had diarrhea for the past 4 days. When assessing the client, the nurse should expect which of the following findings? (Select all that apply.) ___Bradycardia ___Hypotension ___Fever ___Poor skin turgor ___Peripheral edema ___Abdominal cramping

Hypotension Fever Poor skin turgor Abdominal cramping Prolonged diarrhea leads to dehydration, which is characterized by tachycardia, hypotension, fever, lethargy, poor skin turgor, and abdominal cramping. Peripheral edema is more likely to be caused by a fluid overload rather than a fluid deficit. Chapter 45; Page 1096, Box 46-3

A nurse is working at a​ gastroenterologist's office. Which distinction between inflammatory bowel diseases​ (IBD) in the pediatric population should the nurse understand as​ important? Children often present with fistulizing or structuring Crohn disease. IBD is more common in boys than in girls. Most pediatric clients have little colonic involvement with Crohn disease. Surgery is not usually required in the pediatric population.

IBD is more common in boys than in girls. In​ children, more boys than girls have Crohn​ disease, with the opposite true of adults. Pediatric clients with Crohn disease often have ileocolonic or colonic​ disease, where adults do not have any colonic involvement. While children present with inflammatory or nonstricturing or nonfistulizing​ disease, adults are more likely to have strictures or fistulas. IBD tends to be more aggressive in​ children, requiring surgery at a younger age.

What is the first line Tx of IBD exacerbation?

IV steriods corticosteriods/methylprednisolone Antibiotics

The nurse is planning care for a client with inflammatory bowel disease​ (IBD). Which problem should the nurse make a priority for this​ client? Impaired fluid balance Impaired skin integrity Impaired nutrition Fatigue

Impaired fluid balance While all choices are​ problems, impaired fluid balance is a priority that should be addressed in planning care for the client with inflammatory bowel​ disease, because this problem may be life threatening if not addressed.

A client with severe Crohn disease has been on multiple medications without finding adequate relief from the symptoms. Which medication order should the nurse​ anticipate? Methylprednisolone Infliximab Olsalazine Metronidazole

Infliximab Infliximab, an immune response​ modifier, suppresses tumor necrosis factor​ (TNF) to reduce inflammation in the treatment of inflammatory bowel disease. Olsalazine and metronidazole are antibiotics that are used to treat inflammatory bowel disease and do not suppress tumor necrosis factor​ (TNF). Methylprednisolone is a​ corticosteroid, which is used to treat inflammatory bowel disease and does not suppress tumor necrosis factor​ (TNF).

The nurse is planning care for a female client with ulcerative colitis​ (UC). Which characteristic should the nurse​ recognize? (Select all that​ apply.) Vomiting is the predominant manifestation. It affects more women than men. It may include arthritis involving one or several joints. Mild to moderate symptoms includes six or fewer stools per day. Left lower quadrant cramping is relieved by defecation.

It may include arthritis involving one or several joints. Mild to moderate symptoms includes six or fewer stools per day. Left lower quadrant cramping is relieved by defecation. Clients with severe disease may have systemic manifestations such as arthritis involving one or several​ joints, skin and mucous membrane​ lesions, or uveitis​ (inflammation of the​ uvea, the vascular layer of the​ eye, which may involve the sclera and cornea as​ well). Left lower quadrant cramping relieved by defecation is common. Mild to moderate UC is characterized by six or fewer stools per​ day, intermittent rectal bleeding and​ mucus, and few systemic manifestations. It is more common in men than in​ women, with an average age of diagnosis of 40 years.​ Diarrhea, not​ vomiting, is the predominant manifestation of UC.

Is it necessary to give patients more than 5 enemas during an "until clear" order?

No. To reduce the risk of fluid or electrolyte imbalances, do not administer more than three enemas. If the patient seems to not be tolerating the rigor of repeated enemas, notify the health care provider. Chapter 46; Page 1107

"The client diagnosed with appendicitis has undergone an appendectomy. At two hours postoperative, the nurse takes the vital signs and notes T 102.6 F, P 132, R 26, and BP 92/46. Which interventions should the nurse implement? List in order of priority. 1. Increase the IV rate. 2. Notify the health care provider. 3. Elevate the foot of the bed. 4. Check the abdominal dressing. 5. Determine if the IV antibiotics have been administered.

Order of priority: 1, 3, 4, 5, 2." "1. The nurse should increase the IV rate to maintain the circulatory system function until further orders can be obtained. 3. The foot of the bed should be elevated to help treat shock, the symptoms of which include elevated pulse and decreased BP. Those signs and an elevated temperature indicate an infection may be present and the client could be developing septicemia. 4. The dressing should be assessed to determine if bleeding is occurring. 5. The nurse should administer any IV antibiotics ordered after addressing hypovolemia. The nurse will need this information when reporting to the HCP. 2. The HCP should be notified when the nurse has the needed information."

After an enema is infused, what position should the patient be in and why?

Patient should turn from the left lateral to dorsal recumbent, over to the right lateral position. The position change ensures that fluid reaches the large intestine. Chapter 46; Page 1107

A client recovering from a colectomy has the following vital​ signs: ​Pulse: 92​ beats/min Blood​ pressure: 103/76 mmHg ​RR: 18​ breaths/min ​Temperature: 100.1°F Which should be the​ nurse's priority​ action? Documenting the findings as normal Administering acetaminophen as prescribed Performing a focused assessment Informing the healthcare provider

Performing a focused assessment Before taking any​ action, it is important for the nurse to assess the client. The client is at high risk for​ infection, and the rising temperature is a good indication. The nurse should assess the surgical wound site and the skin for signs of breakdown. The nurse needs to act on the elevated​ temperature, not just document the results. Simply administering acetaminophen will not address a possible infection. The nurse should assess the client before informing the healthcare provider so that a complete picture of the​ client's status can be relayed.

"The most common surgical procedures for patients with ulcerative colitis are: 1.Subtotal colectomy and ileostomy. 2.Colostomy and ileo-conduit. 3.Laparoscopic gastrectomy. 4.Segmental resection or

Rationale: Surgery might be necessary for functional older patients with acute disease when drug therapy fails and when multiple precancerous lesions are detected. The most common surgical procedures are subtotal colectomy and ileostomy.

Impaction

Resulting from unrelieved constipation, it is a collection of hardened feces wedged in the rectum that a person cannot expel; fecal impaction. Digital removed and must be careful because of stimulating vagus nerve; bradycardia. Chapter 46; Page 1091

A client with severe ulcerative colitis​ (UC) is prescribed bedrest. Which nursing diagnosis should the nurse identify as a priority for this​ client? Skin​ Integrity, Risk for Impaired Constipation ​Coping, Ineffective Body​ Image, Disturbed

Skin​ Integrity, Risk for Impaired Clients with severe UC may have diarrhea with up to 6 to 10 bowel movements every day. Someone who is also incontinent and prescribed bedrest will be at high risk for impaired skin​ integrity, pressure​ ulcers, and eventually infection.​ Therefore, the priority nursing diagnosis is Skin​ Integrity, Risk for Impaired. ​Coping, Ineffective and Body​ Image, Disturbed may be relevant and important but are not the priority diagnoses. Clients with UC have​ diarrhea, not constipation.

The nurse is preparing a community initiative to help reduce the rate of inflammatory bowel disease​ (IBD). On which behavior should the nurse​ focus? Increasing daily exercise habits Eliminating alcohol use Making lifestyle changes to a​ low-fat diet Smoking cessation

Smoking cessation Smoking cigarettes is a major risk factor for the development of IBD and should be the behavior on which the nurse focuses. Dietary​ changes, such as a​ low-fat diet or eliminating​ alcohol, are not associated with decreasing the risk of IBD. Increasing regular exercise does not reduce the risk of developing IBD.

Laxatives

Soften the stool and promote peristalsis. Chapter 46; Page 1091

What are soapsuds enema?

Stimulating peristalsis through intestinal irritation. USE ONLY PURE CASTILE SOAP because other harsher soap products are contraindicated, as they can cause bowel inflammation. Caution in pregnant women and older adults because they cause electrolyte imbalance or damage the intestinal mucosa. Chapter 46; Page 1107

Effluent

Stool discharged from the stoma. Chapter 46; Page 1109

-otomy

Surgical incision

-ostomy

Surgical opening

Colostomy

Surgical openings are created in the colon. Chapter 46; Page 1093

Ileostomy

Surgical openings are created in the ileum. Chapter 46; Page 1093

Stoma

Temporary or permanent artificial opening. Chapter 46; Page 1093

The nurse is caring for a child with inflammatory bowel disease​ (IBD) and severe diarrhea. Which goal should the nurse identify as a priority for this​ client? The child maintains adequate hydration. The child demonstrates healthy coping skills. The child reports improved sleep. The child​ self-administers prescribed medication.

The child maintains adequate hydration. A child with severe UC is having frequent diarrhea and most likely has a fluid volume deficit. The priority goal for this child is to maintain adequate hydration in order to support​ fluid, electrolyte, and acid-base balance. Healthy coping skills are​ important, but physiological needs should be addressed first. The child may be too young to​ self-administer medication. There is no indication that the child is having difficulty sleeping.

The nurse is preparing for discharge of a client who recieved a colectomy 4 days earlier. Which of the following nursing care goals has the highest priority prior to discharge?"A: Lung are clear per auscultation B: incision is healed without redness or drainage C: bowel sounds present, client expels flatus D: vital signs within normal limits"

The correct answer is C because a collectomy involved the GI tract, the return of normal GI function is most important; the nurse assures the client is expelling gas prior to discharge.

Enema

The installation of a solution into the rectum and sigmoid colon. It is used to promote defecation by stimulating peristalsis. Chapter 46; Page 1107

The nurse is planning care for a client in the early stages of ulcerative colitis. Which part of the colon should the nurse understand to be initially affected by ulcerative​ colitis? The rectosigmoid area The transverse colon The ileocecal junction The duodenum

The rectosigmoid area Ulcerative colitis begins with inflammation at the base of the crypts of​ Lieberkühn in the distal large intestine and rectal mucosa.​ Microscopic, pinpoint mucosal hemorrhages​ occur, and crypt abscesses develop. These abscesses penetrate the superficial submucosa and spread​ laterally, leading to necrosis and sloughing of bowel mucosa. Ulcerative colitis can progress to the entire​ colon, stopping at the ileocecal junction. The duodenum is part of the small intestine and is not affected by ulcerative colitis. The transverse colon is not initially affected by ulcerative colitis.

A client is admitted with irritable bowel syndrome. The nurse would anticipate the client's history to reflect which of the following? 1. Pattern of alternating diarrhea and constipation. 2. Chronic diarrhea stools occurring 10-12 times per day. 3. Diarrhea and vomiting with severe abdominal distention. 4. Bloody stools with increased cramping after eating.

Think about each answer choice. (1) correct-condition is often called spastic bowel disease; no inflammation is present (2) refers to inflammatory bowel disease such as ulcerative colitis or Crohn's disease (3) refers to inflammatory bowel disease such as ulcerative colitis or Crohn's disease (4) bloody stools do not occur 38.

A client with inflammatory bowel disease is prescribed sulfasalazine. Which information should the nurse instruct the client about this​ medication? (Select all that​ apply.) It reduces the number of stools per day to maintain fluid balance. Use sunscreen when taking this. The main side effect is appetite stimulation. It blocks the production of prostaglandin to reduce inflammation. Reduce intake of sodium when taking this.

Use sunscreen when taking this. It blocks the production of prostaglandin to reduce inflammation. Sulfasalazine blocks the production of prostaglandin to reduce inflammation. The client should use sunscreen when taking this medication because it causes photosensitivity. This medication does not stimulate appetite or reduce the number of stools per day. Clients taking​ corticosteroids, not​ antibiotics, should decrease sodium intake. This medication is an​ antibiotic, which does not require reducing sodium intake.

Constipation

a condition in which there is difficulty in emptying the bowels, usually associated with hardened feces. Chapter 46; Page 1091

Which statements does the nurse include while providing discharge instructions for a patient with giardiasis? Select all that apply. a. "Avoid contact with stool from dogs and beavers." b. "All household and sexual partners should have stool examinations for parasites." c. "Treatment will most likely consist of metronidazole (Flagyl)." d. "The infection can be transmitted to others until the amebicides kill the parasites." e. "Stools are examined 6 days after treatment to assess for eradication."

a. "Avoid contact with stool from dogs and beavers." b. "All household and sexual partners should have stool examinations for parasites." c. "Treatment will most likely consist of metronidazole (Flagyl)." d. "The infection can be transmitted to others until the amebicides kill the parasites."

A patient with UC who has had an ileostomy is being discharged home. The nurse has provided discharge teaching. Which statements by the patient indicate the teaching has been effective? Select all that apply. a. "I will avoid foods that cause gas." b. "I will call the health care provider if I have a fever over 101 degrees F." c. "I will change the adhesive for the appliance daily." d. "I know the pouch needs emptying when I feel pain in that area." e. "I will call the health care provider if I feel like my heart is beating fast."

a. "I will avoid foods that cause gas." b. "I will call the health care provider if I have a fever over 101 degrees F." e. "I will call the health care provider if I feel like my heart is beating fast."

A patient is prescribed sulfasalazine (Azulfidine) for the treatment of UC. Which patient statement indicates the patient is experiencing a side effect of this drug? a. "My skin is covered with a rash." b. "My knees hurt." c. "My appetite has increased." d. "I wake up at night sweating sometimes."

a. "My skin is covered with a rash."

The respiratory problems that may accompany peritonitis are a result of which factor? a. Associated pain interfering with ventilation b. Decreased pressure against the diaphragm c. Fluid shifts to the thoracic cavity d. Decreased oxygen demands related to the infectious process

a. Associated pain interfering with ventilation

Which interventions are useful in preventing spread of gastroenteritis? Select all that apply. a. Careful hand washing b. Sanitize all surfaces that may be contaminated c. Prophylactic use of antibiotics d. Easily accessible hand sanitizers e. Test all food preparation employees

a. Careful hand washing b. Sanitize all surfaces that may be contaminated d. Easily accessible hand sanitizers

A patient is suspected to have ulcerative colitis (UC). Which definitive diagnostic test does the nurse expect the patient to undergo in order to confirm the diagnosis? a. Colonoscopy b. C-reactive protein c. Albumin levels d. Erythrocyte sedimentation rate

a. Colonoscopy

What is the nature of pain associated with diverticulitis? a. Intermittent becoming progressively steady b. Sharp and continuous c. Localized to the right upper quadrant d. Severe and incapacitating

a. Intermittent becoming progressively steady

The nurse is caring for the patient with acute appendicitis. Which interventions will the nurse perform? Select all that apply. a. Maintain the patient on NPO status. b. Administer IV fluids as prescribed. c. Apply warm compresses to the right lower abdominal quadrant. d. Maintain the patient in the supine position. e. Administer laxatives.

a. Maintain the patient on NPO status. b. Administer IV fluids as prescribed.

Which intervention does the nurse delegate to the unlicensed assistive personnel (UAP) when caring for a postoperative patient with peritonitis? a. Measure intake and output. b. Assess wound drainage. c. Administer IV antibiotics. d. Teach patient about wound care.

a. Measure intake and output.

Which statement about diverticular disease is true? a. Most diverticula occur in the sigmoid colon. b. Diverticula are uncomfortable even when not inflamed. c. High-fiber diets contribute to diverticula occurrence. d. Diverticula form where intestinal wall muscles are weak.

a. Most diverticula occur in the sigmoid colon.

Which description best defines an anal fissure? a. Perianal tear that can be very painful b. Duct obstruction and infection c. Communicating tract d. Localized are of induration with pus

a. Perianal tear that can be very painful

In caring for a patient with CD, the nurse observes for which complications? Select all that apply. a. Peritonitis b. Small bowel obstruction c. Nutritional and fluid imbalances d. Presence of fistulas e. Appendicitis f. Severe nausea and vomiting

a. Peritonitis b. Small bowel obstruction c. Nutritional and fluid imbalances d. Presence of fistulas

Which statements about peritonitis are true? Select all that apply. a. Peritonitis is caused by contamination of the peritoneal cavity by bacteria or chemicals. b. Continuous ambulatory peritoneal dialysis (CAPD) can cause peritonitis. c. White blood cell counts are often decreased with peritonitis. d. Abdominal wall rigidity is a classic finding in patients with peritonitis. e. Chemical peritonitis is caused by leaking of pancreatic enzymes or gastric acids.

a. Peritonitis is caused by contamination of the peritoneal cavity by bacteria or chemicals. b. Continuous ambulatory peritoneal dialysis (CAD) can cause peritonitis. d. Abdominal wall rigidity is a classic finding in patients with peritonitis.

The ED nurse is assessing a patient admitted with frequent, liquid, foul-smelling stools containing mucus and blood. Assessment findings include temperature 103.8 F, tenesmus, abdominal tenderness, and vomiting. Which additional laboratory tests does the nurse expect to collect? a. Serial stool samples b. Urine culture c. Throat culture d. Sputum culture

a. Serial stool samples

The nurse is admitting a client with the diagnosis of appendicitis to the surgical unit. Which question is essential to ask? A."When did you last eat?" B."Have you had surgery before?" C."Have you ever had this type of pain before?" D."What do you usually take to relieve your pain?"

answer A. When a person is admitted with possible appendicitis, the nurse should anticipate surgery. It will be important to know when she last ate when considering the type of anesthesia so that the chance of aspiration can be minimized. The other inoformation is "nice to know", but not essential.

Which of the nursing interventions should be implemented to manage appendicitis? a. Assess pain b. encourage oral intake of clear fluids. c. provide discharge teaching D. assess for symptoms of peritonitis.

answer D. Monitor for peritonitis because if the appendix ruptures, bacteria can enter the peritoneum. Pain will be managed with analgesics, and pt should be NPO for surgery. Discharge is not done at this time

The nurse is providing teaching for a patient with an anal fissure as a complication of CD. Which statement by the patient indicates the need for further teaching? a. "I will use warm sitz baths." b. "A diet that is low in bulk-producing agents is best for me." c. "Hydrocortisone cream may be helpful to decrease comfort." d. "Topical anti-inflammatory agents will help if I am uncomfortable."

b. "A diet that is low in bulk-producing agents is best for me."

The patient with gastroenteritis due to infection with the norovirus asks the nurse how this illness occurred. Which statement by the patient indicates correct understanding of the nurse's teaching? a. "I got this infection from being around my grandchildren when they had respiratory illnesses." b. "It is likely that I got this illness from either contaminated water or food." c. "I may have gotten sick when I was traveling last month." d. "It's really important that everything I eat is cooked until it is well done."

b. "It is likely that I got this illness from either contaminated water or food."

Which are common manifestations in a 28-year-old patient with dehydration secondary to gastroenteritis? Select all that apply. a. Peripheral edema b. Elevated temperature c. Dry mucous membranes d. Hypertension e. Oliguria

b. Elevated temperature c. Dry mucous membranes e. Oliguria

Which characteristics pertain to Crohn's disease (CD)? Select all that apply. a. Begins in the rectum and proceeds in a continuous manner toward the cecum b. Fistulas commonly develop c. Five to six soft, loose stools per day that are nonbloody d. Increased risk of colon cancer e. Some patients experience extra intestinal manifestations such as migratory polyarthritis, ankylosing spondylitis, and erythema nodosum f. Cobblestone appearance of the internal intestine

b. Fistulas commonly develop c. Five to six soft, loose stools per day that are nonbloody f. Cobblestone appearance of the internal intestine

What is the priority nursing concern for a patient with gastroenteritis? a. Nutrition therapy b. Fluid replacement c. Skin care d. Drug therapy

b. Fluid replacement

Which interventions does the nurse expect to implement when caring for a patient with diverticulitis? Select all that apply. a. Laxative and enemas as ordered b. IV fluids to prevent dehydration c. Broad-spectrum antibiotics d. Teach the patient to refrain from lifting or straining e. Keep the patient NPO if symptoms are severe

b. IV fluids to prevent dehydration c. Broad-spectrum antibiotics d. Teach the patient to refrain from lifting or straining e. Keep the patient NPO if symptoms are severe

Which statement is true about the medical treatment of UC? a. Infliximab (Remicade) is approved as a first-line therapy. b. Immunomodulators are not thought to be effective; however, in combination with steroids, they may offer a synergistic effect. c. When a therapeutic level of glucocorticoids is reached, the dosage of the drug stays the same to maintain the therapeutic effect. d. The method of action for the aminosalicylates is interruption of the pain pathway.

b. Immunomodulators are not though to be effective; however, in combination with steroids, they may offer a synergistic effect.

Which nursing intervention is part of nonsurgical management for a patient with peritonitis? a. Monitor weekly weight and intake and output. b. Insert a nasogastric tube to decompress the stomach. c. Order a breakfast tray when the patient is hungry. d. Administer NSAIDs for pain.

b. Insert a nasogastric tube to decompress the stomach.

The patient has been diagnosed with acute appendicitis. Based on this diagnosis, which interventions does the nurse perform? a. Start a bowel cleansing program. b. Prepare the patient for surgery. c. Apply a heating pad to the lower abdomen. d. Assess the patient's knowledge about dietary modifications.

b. Prepare the patient for surgery.

The nurse on the surgical unit is expecting to admit the patient who has had an appendectomy with abscess. What does the nurse anticipate care for this patient will include? Select all that apply. a. Clear liquids b. Wound drains c. IV antibiotics d. Nonsteroidal anti-inflammatory drugs (NSAIDs) for pain control e. Nasogastric (NG) tube care

b. Wound drains c. IV antibiotics e. Nasogastric (NG) tube care

The nurse is instructing a patient about home care after an exploratory laparotomy for peritonitis. Which statement by the patient indicates that teaching has been effective? a. "It is normal for the incision site to be warm." b. "I will stop taking the antibiotics if diarrhea develops." c. "I will call the health care provider for a temperature greater than 101 degrees F." d. "I will resume activity with my bowling league this week for exercise."

c. "I will call the health care provider for a temperature greater than 101 degrees F."

What are the cardinal signs of peritonitis? a. Fever and headache b. Dizziness with nausea and vomiting c. Abdominal pain, distention, and tenderness d. Nausea and loss of appetite

c. Abdominal pain, distention, and tenderness

The patient comes to the emergency department (ED) with right lower quadrant pain. What does the ED nurse suspect? a. Gastroenteritis b. Ulcerative colitis c. Appendicitis d. Crohn's disease

c. Appendicitis

Which type of stoma will a patient with diverticulitis most likely have postoperatively? a. Ileostomy b. Jejunostomy c. Colostomy d. Cecostomy

c. Colostomy

The fluid shift that occurs in peritonitis may result in which of the following? a. Intracellular fluid moving into the peritoneal cavity b. Significant increase in circulatory volume c. Decreased circulatory volume and hypovolemic shock d. Increased bowel motility caused by increased fluid volume

c. Decreased circulatory volume and hypovolemic shock

Which parasitic infection is manifested by diarrhea and occurs most commonly in immunosuppressed patients, especially those with human immunodeficiency virus (HIV)? a. Entamoeba histolytica b. Cryptosporidium c. Giardia lamblia d. Escherichia coli

c. Giardia lamblia

Which statement is true about drug therapy for CD? a. Budesonide (Entocort EC) is a rapid-release compound that delivers low local glucocorticoid concentrations to the terminal ileum for patients with CD. b. Methotrexate (Rheumatrex) is contraindicated in the treatment of CD. c. Metronidazole (Flagyl) has been helpful in patients with fistulas and CD. d. Adalimumab (Humira) is a glucocorticoid approved for the treatment of CD.

c. Metronidazole (Flagyl) has been helpful in patients with fistulas and CD.

Which is a preventive measure for diverticular disease? a. Excluding whole-grain breads from the diet b. Avoiding fresh apples, broccoli, and lettuce c. Taking bulk agents such as psyllium hydrophilic mucilloid (Metamucil) d. Taking routine anticholinergics to reduce bowel spasma

c. Taking bulk agents such as psyllium hydrophilic mucilloid (Metamucil)

The nurse is caring for a patient with gastroenteritis who has frequent stools. Which task is best to delegate to the UAP? a. Teach the patient to avoid toilet paper and harsh soaps. b. Instruct the patient on how to take a sitz bath. c. Use a warm washcloth to remove stool from the skin. d. Dry the skin with absorbent cotton.

c. Use a warm washcloth to remove stool from the skin.

As part of the routine treatment plan for a patient with bacterial gastroenteritis, which drugs does the nurse anticipate the patient will most likely be prescribed? a. Anticholinergics b. Antiemetics c. Antiperistaltic drugs d. Antibiotics

d. Antibiotics

A patient with CD has a fistula. Which assessment finding indicated possible dehydration? a. Weight gain of 2 pounds in one day b. Abdominal pain c. Foul-smelling urine d. Decreased urinary output

d. Decreased urinary output

Which laboratory finding does the nurse expect may occur with a diagnosis of appendicitis? a. Decreased hematocrit and hemoglobin b. Increased coagulation time c. Decreased potassium d. Increased WBC count

d. Increased white blood count

Which type of diet has been implicated in the formation of diverticula? a. High-fat diet b. Low-protein diet c. High-cholesterol diet d. Low-fiber diet

d. Low-fiber diet

Which drug is often used in older patients for pain management of moderate to severe diverticulitis? a. Ibuprofen (Motrin) b. Acetaminophen (Tylenol) c. Aspirin (Anacin) d. Morphine sulfate (Duramorph)

d. Morphine sulfate (Duramorph)

The nurse is assessing an older adult patient with abdominal pain. Assessment findings include generalized abdominal pain with rigidity, nausea and vomiting, elevated temperature (101.2 F), increased heart rate (122/minute) and chills. The patient is also somewhat confused and does not know where he is. What does the nurse suspect with this patient? a. Crohn's disease b. Ulcerative colitis c. Diverticulitis d. Peritonitis

d. Peritonitis

The nurse is assessing a patient with viral gastroenteritis. Which symptom is the nurse most concerned about? a. Orthostatic blood pressure changes b. Poor skin turgor c. Dry mucous membranes d. Rebound tenderness

d. Rebound tenderness

Which surgical procedure involves removal of the colon, rectum, and anus with the surgical closure of the anus? a. Restorative proctocolectomy with oleo pouch-anal anastomosis (RPC-IPAA) b. Natural orifice trans luminal endoscopic surgery (NOTES) c. Sigmoid colostomy d. Total proctocolectomy with a permanent ileostomy

d. Total proctocolectomy with a permanent ileostomy

The nurse notes that a pediatric client with inflammatory bowel disease​ (IBD) has had poor growth since the last examination. Which suggestion should the nurse make to the​ client's parents? ​"Add daily probiotic​ supplements." ​"Administer liquid dietary​ supplements." ​"Increase fiber intake to add bulk to​ stools." ​"Decrease fluid intake to reduce​ diarrhea."

​"Administer liquid dietary​ supplements." A child with IBD is at higher risk for poor growth and malabsorption. The nurse should advise the parents to increase calorie and protein intake. Liquid dietary supplements can be very effective to promote growth. Adding fiber or probiotics and decreasing fluid intake will not promote growth and nutrition status in this child.

The parent of a child with inflammatory bowel disease​ (IBD) reports that the child is refusing to​ eat, causing the parent to become frustrated. Which advice should the nurse provide to this​ parent? ​"It is important for the child to eat. Make foods that they​ like." ​"Be firm. The child will eat when​ hungry." ​"I'll let the healthcare provider know about your​ concerns." ​"I'll arrange for a consult with a nutritionist to get more ideas for meal​ time."

​"I'll arrange for a consult with a nutritionist to get more ideas for meal​ time." It is important for the parents to avoid making mealtime a source of family struggle or difficulty. Referral to a nutritionist or dietitian can help the family learn new​ food/meal strategies. Telling the parent to be firm only reinforces the conflict. Simply making the food that the child likes may not be​ appropriate, because it may worsen symptoms. Informing the healthcare provider may be​ necessary, but it does not address the​ family's concerns.

A client with inflammatory bowel disease​ (IBD) is prescribed corticosteroids. Which information should the nurse provide about the​ medications? (Select all that​ apply.) ​"Take with food or at​ mealtimes." ​"Expect weight​ loss." ​"It may cause low blood​ pressure." ​"Reduce intake of foods high in​ sodium." ​"Take medication consistently and do not stop​ abruptly."

​"Take with food or at​ mealtimes." ​"Reduce intake of foods high in​ sodium." ​"Take medication consistently and do not stop​ abruptly." Corticosteroids should be taken with food or at mealtimes to reduce the gastrointestinal side effects. A client taking corticosteroids tends to retain​ fluid, and sodium tends to exacerbate the retention.​ Therefore, the client should reduce the intake of foods that are high in sodium. Corticosteroids should be taken consistently and not be stopped abruptly to reduce the possibility of adrenal shutdown. A client taking corticosteroids tends to​ gain, not​ lose, weight. A client taking corticosteroids tends to have​ high, not​ low, blood pressure.

A newly pregnant client who has a history of inflammatory bowel disease​ (IBD) asks how the disease will impact their pregnancy. Which response by the nurse is accurate​? ​"You will need to be on bedrest throughout the entire​ pregnancy." ​"Your disease may increase the risk for some pregnancy​ complications." ​"The medications used to treat your disease are safe to use during your​ pregnancy." ​"IBD should have no effect on pregnancy or the baby.​ We'll watch you​ closely."

​"Your disease may increase the risk for some pregnancy​ complications." Pregnant women with IBD are at higher risk for​ preeclampsia, medically indicated preterm​ delivery, preterm premature rupture of​ membranes, and delivering infants with low APGAR scores. There is also a risk of some major congenital​ malformations, which are not completely explained by the medications used. It is not necessary for a woman to be on complete bedrest throughout her entire pregnancy. Not all drugs are safe for the pregnant woman.


Kaugnay na mga set ng pag-aaral

West Coast EMT Chapter 17 -- Neurologic Emergencies

View Set

Section 23D ceremonies and events

View Set

ACCT Decision-making & incremental analysis

View Set

final exam review for intro to business

View Set

Macroeconomics Review Questions # 1

View Set

Managing Family Business Chapter 3

View Set

Chapter 14: Firms in Competitive Markets

View Set